Этого треда уже нет.
Это копия, сохраненная 16 июля 2017 года.

Скачать тред: только с превью, с превью и прикрепленными файлами.
Второй вариант может долго скачиваться. Файлы будут только в живых или недавно утонувших тредах. Подробнее

Если вам полезен архив М.Двача, пожертвуйте на оплату сервера.
285 Кб, 2560x1440
36 Кб, 487x600
Тред тупых вопросов #61 #316443 В конец треда | Веб
Тред вопросов о жизни, Вселенной и всем таком.

Спрашиваем то, за что в других местах выдают путёвку в биореактор. Здесь анонимные ученые мирового уровня критически рассмотрят любые гениальные идеи и нарисованные в Paint схемы.

Предыдущий обратился в излучение Хокинга
https://2ch.hk/spc/res/315296.html (М)
#2 #316445
>>316440

>можно что угодно говорить и придумывать. человек это не может увидеть или проверить собственными глазами. скажешь звезд миллион, будет миллион, скажешь миллиард, будет считаться миллиард. не веришь проверь сам, пересчитай. скажешь есть ось галактики и ее крутит чупакабра, все вынуждены поверить на слово



ох лоооол
#3 #316450
Что думают аноны по поводу того, что большой взрыв был создан в симуляции? Создавалась ли матрица исключительно для нас, или просто так удачно совпали обстоятельства, что появилась жизнь на Земле, а далее люди?
Какие можете привести аргументы "за"и "против" данной теории, основываясь на реальных научных данных, без фантастики?
#4 #316451
>>316450
Реальные научные данные опровергают данную теорию и мне лень объяснять даже как. Если интересно, то сам ищи. Так что пока только иди на хуй вот почему.
Надеюсь мой ответ поможет.
316473316482
#5 #316457
>>316450

>просто так удачно совпали обстоятельства, что появилась жизнь на Земле, а далее люди?


Ну просто типа кароч оно само так хуяк - и из звездной пыли родились динозавры, ну ты понел.
sage #6 #316463
>>316443 (OP)
У Сагана 9-го ДР был, а ты какого-то Митю Каканого прилепил на оп-пик и манякартинку, оп-ты хуй.
#7 #316473
>>316451
Лол. Илан Маск даже эту теорию задвигал же.
316482
#8 #316477
>>316450
Если ты имеешь в виду гипотезу симуляции Вселенной, то это не наука вообще, ибо в общем смысле не фальсифицируема. Это к философии больше. Было несколько соображений, как это проверить, приняв определенные допущения - что вычислительные ресурсы этого "компьютера" не бесконечны и т.п., но они все шиты белыми нитками, мягко говоря.
316484316487
#9 #316482
>>316450

>данной теории


>>316451

>опровергают данную теорию


>>316473

>теорию


Мне начинает казаться, что даже в этом треде за неуместное употребеление слова "теория" необходимо выписывать направление в биореактор.
316483
32 Кб, 378x363
#10 #316483
>>316482

>за неуместное употребеление слова "теория" необходимо выписывать направление в биореактор


Два чаю.
#11 #316484
>>316477
Ну это допущение не заслуживает никакой критики и учёные cерьёзно это не воспринимают. Законы квантовой механики: "объекты существуют только когда ты на них смотришь" неплохо реализовано на новых видеокартах nvidia с помощью DX12. Так что, существование матрицы доказано что ле? Бред же.
#12 #316487
>>316477

>это не наука вообще, ибо в общем смысле не фальсифицируема


Этот товаризч прав.

>просто так удачно совпали обстоятельства, что появилась жизнь на Земле, а далее люди?


https://ru.wikipedia.org/wiki/Антропный_принцип
#13 #316491
>>316479

>То есть мы постоянно удаляемся от других галактик?


Да. Кстати, мы не приближаемся к Великому Аттрактору, а удаляемся от него. Медленнее, чем от остальных, но всё же удаляемся. Расширение пространства сильнее гравитационных сил Аттрактора и Сверхскопления Шепли.
316493
#14 #316493
>>316491
А нам не похуй? От солнца же мы не удаляемся. Да и внутри самой галактики есть чего заселять.
316494316496
#15 #316494
>>316493
Лол, ебать ты тупой
В конце концов и атомы распадутся.
316495316557
#16 #316495
>>316494
К тому времени мы научимся их собирать. Или перенесем свои сознания в атсрал.
316497
#17 #316496
>>316493
Если найдём червоточины, то нам и на расширение галактики похуй.
#18 #316497
>>316495
К тому времени лысые обезьяны эволюционируют во что-то большее, чем просто кучка костей, мяса и воды.
316498316499
#19 #316498
>>316497
Это уже после ядерной войны или до?
316499
#20 #316499
>>316497
>>316498
В /sf, животные
316500
#21 #316500
>>316499
А что тут такого фантастического? Это вполне подпадает под футурологию. Или ты отрицаешь эволюцию?
316501316504
#22 #316501
>>316500
Не стоит так просто распространять эволюцию на что-то большее, чем мясные мешки с костями.
#23 #316502
У вас опять /б протек.
#24 #316504
>>316500
Вот не надо у нас тут выдавать футррологию за науку, уважаемый. С маняфантазиями в сайфай пройдите.
#25 #316557
>>316494
Слышь пидр, через 10000 лет время пойдет вспять, а с ней и расширение вселенной, как тебе такая теория, питух?
#26 #316576
>>316443 (OP)
Спейсаны, помогите понять как решать такие задачи.

>Астрометрический спутник движется по орбите Юпитера. По его измерениям за 6 лет наблюдений положение звезды относительно объектов далекого фона изменилось примерно на 0,01 угловой секунды. Каково расстояние до этой звезды в парсеках?



По формуле расстояние в парсеках находится так
Dp = Rau/arcsec(x)
Dp - расстояние в парсеках
Rau - расстояние от центра Солнца до центра планеты в астрономических единицах (у Юпитера он приблизительно в 5 раз больше, чем у Земли)
arcsec(x) - параллакс
то есть получается в данном случае так
Dp = 5 / arcsec(0.01) = -0.9437
Очевидно, результат с правильным ответом не сходится, мягко говоря. Как вы поняли я совершенный нуб в тригонометрии и математике вообще.
ЧЯДНТ?
316656316670
#27 #316580
Хм... Если скорость света ограничена массой, получается что частица с отрицательной массой может путешествовать быстрее?
316581316582316596
#28 #316581
>>316580
нет. она всего лишь будет двигаться по оси времени в обратную сторону
316596
#29 #316582
>>316580
нет, нужна масса в мнимых числах
#30 #316596
>>316580
>>316581

>частица с отрицательной массой


Должна будет иметь отрицательную скорость. Постойте-ка, скорость - вектор. Так что нихуя не будет.
316600
#31 #316600
>>316596
это тебе с твоей точки зрения так кажется, а на деле она будет лететь не назад, а вперед, но в обратную сторону во времени.
316625
#32 #316621
Хочу запилить ракету. Хотя бы на твердом топливе.
Но делать говно на сахаре не хочу: хуй запилишь что-то с предсказуемыми характеристиками.
В США это решается просто: заказываешь шашки хоть в форме Аллаха, и делаеш свой SRB.
Посоветуйте смесь, чтобы более-менее легко достатать компоненты, не взорваться при готовке, и, в идеале, не иметь проблем с ментами за изготовление ВВ.
316623316625316627
#33 #316623
>>316621

> не иметь проблем с ментами за изготовление ВВ.


нет пути, просто не привлекай внимания
#34 #316625
>>316600
Ты ёбнутый? Тебе русским по белому написали, что масса для этого должна быть мнимой.
>>316621
ЧП
316626316653
#35 #316626
>>316625

>Тебе русским по белому написали, что масса для этого должна быть мнимой


Ладно, я погорячился, читал жопой.
#36 #316627
>>316621
Если ты именно про сахарозу, то классический 65/35 сорбит+нитрат калия гораздо более стабильны, чем сахар. Но нужен именно сорбит, а не что-то с чем-то, и термостат и вибростол для плавления и перемешивания смеси, и можно заливать собственные "блинчики" очень стабильного состава и консистенции. Процедура простая и при соблюдении ТБ не более безопасная, чем кручение своих патронов - там просто нечему выйти из-под контроля кроме термостата. Во всяком случае намного более безопасная, чем все остальное. Потом из этих сегментов набираешь ракету сколько нужно. И это не ВВ. Моделисты так и пускают обычно.
#37 #316648
Кароч, что будет если Солнца потеряет массу резко и она уйдет в пустоту? Закон сохранения энергии полетит в ебеня, это очевидно, т.к. система незамкнутая. Как изменится большая полуось земной орбиты если Солнце потеряет, например 1% массы
#38 #316652
Давайте представим: элеватор таки построили.
Почему он должен простоять хотя-бы 10 лет и не упасть, под воздействием нарушения структуры этих злоебучих нанотрубок, на молекулярном уровне микрометеорами и быстрых частиц или банальными микробами вроде тех кто сожрал "Мир"?
316654316655316674
#39 #316653
>>316625
Материя с отрицательной массой будет обладать антигравитационным свойством?
316654316672
#40 #316654
>>316653
отрицательной что?
>>316652
потому что перед постройкой решат все подобные проблемы, либо не будут строить, пока не решат
316696
#41 #316655
>>316652
"мир" никто не сожрал его утопили в океане
316658
#42 #316656
>>316576
Бамп. Слишком сложно для этого треда?
#43 #316658
>>316655
А элеватор утопят на чей-то дом.
#44 #316670
>>316576

>arcsec(0.01)


Ээм, а как ты это считал? Просто 0.01 секунды это ведь очень малое значение, 1/360000-я градуса. И вообще подозрительная сука задача, в лучших традициях методики обучения не понятно чему. 6 земных лет? Ну это половина оборота Юпитера вокруг Солнца. И почему тогда в формуле расстояние до Солнца, если в таком случае база будет равна двум таким расстояниям? Олсо откуда такая перегруженная формула? Я блять этот арксеканс на калькуляторе то не могу посчитать ни в радианах, ни в градусах. Пишет значение не подходит. Ну тут может сам где объебался. По стандартной формуле у меня вышло ~500, если брать базу за 5 а. е. и 1000, если соответственно 10.
316687316688
#45 #316672
#46 #316674
>>316652
Ну это вопрос из разряда "допустим лифт в высотном небоскребе таки построили. Почему он должен возить людей хотя бы 10 лет и не ебнуться в шахту, под воздействием окисления стального тросса, под воздействием трения на шарнири и подшипники или банальными хедкрабами проде тех, что сожрали мозг техника Петра."
Ответ - сервис и разумное конструирование + проектирование. Само по себе наличие лифта позволит проводить ремонт любых износившихся/поврежденных участков очевидным подъемом к этим участкам. Причем куда более безопасным, чем пинками на петардах, полных гидразина. Конструирование с запасом прочности, резервными несущими нагрузку элементами и вот уже практически невероятное попадание микрометеорита точно в цель будет лишь вопросом ремонта, а не трагедии. Крупные каменюки могут стать проблемой, но опять же с лифтом на уровень этой каменюки можно вывести йоба лазер-ган или уж на худой конец хлопушку с тротилом. Но это все опять вопросы уровня, а что делать с лифтом в небоскребе, если в борт здания вдруг въебется самолет. Про быстрые частицы вообще пушка. Да уж скорее эйфелева башня рухнет от выветривания опор, чем космическая радиация наебнет космический лифт. В общем-то псевдовакуум в околоземной сфере - куда менее агрессивная среда для любой металлоконструкции, да и просто конструкции, нежели наша земная поверхность. На поверхности перепад температур, непостоянность влаги, давления воздуха, химическая/биологическая среда, подвергающая коррозии. В космосе нет этих проблем, реальная проблема, которую ты назвал - микрометеориты. Они будут "истирать" конструкцию, но блять 10 лет? !0 лет - слишком малый срок, чтобы сломать лифт нахуй, а с должным сервисом он вполне может простоять и несколько веков. Да и падение ты представляешь видимо как падение гигантского небоскреба, вот только с точки зрения орбитальной механики даже в случае разрыва все будет не так однозначно. Тут скорее трагедией будет проеб гигантских средств, чем проеб тысяч жизней.
316709316781
#47 #316687
>>316670

>и 1000, если соответственно 10.


Вот правильный ответ 1000. Как ты это посчитал?
316690
#48 #316688
>>316670

>Ээм, а как ты это считал?


На вольфрамальфе.
6 Кб, 319x167
#49 #316690
>>316687
L - база, расстояние между двумя точками наблюдения. При периоде вращения в ~11.8 лет, 6 лет как раз будет сроком прохождения Юпитером из одной точки до противоположной ей на орбите, если не вдаваться в эллиптичности орбит. То есть база будет равна диаметру орбиты Юпитера ~10.4 а. е.
Округлить до десяти, поделить на 0.01″ в радианах. 0.01″ в радианах будет 1/360000 и еще разделить на 57.3. 10 делим на эту хуйню с кучей нулей, получаем расстояние
~206280000 в астрономических единицах. В одном парсеке 206 265 а. е., столько же, сколько угловых секунд в одном радианевот тут насчет падежа и родовой принадлежности не уверен, лол. В результате деления получим 1000 Пк с копейками.
В общем если тебе препод дал эту формулу, то разбирайся с ним, выпрашивай пояснений. Меня эти обратные тригонометрические всегда отпугивали, тем более арксеканс. Просто в случае параллакса геометрия, на мой взгляд, довольно очевидная и простая, поэтому и формула из википедии подойдет.
316717316935
#50 #316696
>>316654

>не будут строить, пока не решат


Т.е. никогда не будут строить. Ввиду концептуальности проблемы.
Или возможно от микробов лифт защитить?
#51 #316709
>>316674
Да он не упадёт как башня. Но куча кусков явно будет сыпаться в рандомном направлении. И когда я говорил о рассыпании имелись ввиду именно микробы которые через 10 лет акурат заполонят весь свободный объём башни. Что едят микробы в космосе? - Всё. В том числе нанотрубки. что случится с лифтом когда его планомерно начнут кушать? Ты предлагаешь заменять части лифта, но просто так не поменяешь целиком кусок.
Да с временем проебался, но время эксплуатации по сравнению с ценой на строительство слишком мало. Если и построят его, это будет эдак лет через 200 когда лифт будет стоить, хотя бы, 10 летнему бюджету космического агенства Н.
316710316716
#52 #316710
>>316709
Так стерилизовать надо.
#53 #316716
>>316709

> Что едят микробы в космосе? - Всё


уёбывай со своим рентв
#54 #316717
>>316690
Cпасибо, братюнь. Теперь понятно. Это задача из онлайн-курса по астрономии, который сейчас на lektorium.tv идёт - лектор Владимир Сурдин.
#55 #316719
Почему посадку евромарсохода доверили Лавке? Они там ебанулись, или просто хотят замести следы распила с помощью неудачной посадки?
316912
#56 #316757
1.А если при падении в чёрную дуру и взгляде вовне проходит почти вечность вселенной, то и сама чд на глазах стремительно испаряется, и когда чёрный анус сингулярности почти тебя поглотил и тебя порядком распидорасило на кванты, ГС уже столь близко от тебя, что принцип неопределённости решает и ты испаряешься вовне прямо на своих глазах, а потом и вся чд вырождается в планковскую, в итоге проблема сингулярности решена - в неё просто ничего не успевает попасть, - это антинаучная хуйня всё?
2. Можно ли испарение интерпретировать как туннелирование падающего вещества, которое со стороны наблюдателя "заморожено" на горизонте событий (в силу красного смещения и замедления времени)? Ну т.е. энергия забирается не прямо из сингулярности, а с горизонта.
316760316767
#57 #316760
>>316757
1. Ну, касательно, проблемы сингулярности так и есть, собственно-говоря, для стороннего наблюдателя. А для объекта погружения в ЧД вечность не проходит, он там в своем времени движется или хуй знает как там у него с
восприятием.
2.Так она с горизонта и так забирается.
316761
#58 #316761
>>316760
ненене, нигра по дискавери грил, если падающий смотрит вовне, то он будет видеть внешнюю вселенную во всё более ускоряющемся времени и даже смерть аллаха увидит. По ОТО так.
#59 #316766
Блять, вы на основаниее одной сраной системы уравнений, которая (и так уже известно) не для всего наблюдаемого работает, пытаетесь че-то там предасказать для областей с запредельной гравитацией. Вы сами понимаете что это просто сайфай?
#60 #316767
>>316757
Сто раз уже объясняли блядь.
Ничего ты не увидишь ибо свет не дойдет до глаз.
316768
12 Кб, 200x306
#61 #316768
>>316767
Объясняешь про сайфай, ебать ты молодец. Зайду в тред любителей властелина колец, так мне там также "100 раз объяснят" кто из орков выебал Арагорна. На основании одной книжки. Ты блять эксперименты проводил чтоли?
37 Кб, 522x392
#62 #316771
поясните как происходит взаимодействие электрона с ядром. электрон с ядром обменивается фотонами. почему фотоны летают м-ду электроном и ядром, а не улетают в ебеня? с какой частотой происходит обмен фотонами?
316787
#63 #316781
Чет давненько я у вас не был. Ответил на интересные для себя вопросы.

>>315817
>>315818
Тащемта почти на всех языках название нашей планеты примерно означает "грязь". Ну, как земля. Когда употребляется не в контексте "суша" как противоположность морям-океанам

>>315996
Любая ЧД распидорасит. Малые спагеттифицируют, а большие позволят добраться до ГС, который уже сам распидорасит.
Я думал, что всем очевидно, что горизонт событий - гроб гроб кладбище пидор и одним куском его не пересечь.

>>316178
У нас полно радиотелескопов.

>>316341
Если не будет говна в атмосфере намешано, то синий из-за рэлеевского рассеяния, т.к. в спектре любой звезды есть синий цвет.

>>316367
Когда грязища подусядется, то будет такой же синий из-за того же рэлеевского рассеяния.

>>316674
Если отстрелить противовес то лифт очень брутально намотается на экватор приведя к потере большого количества человеческих жертв и уничтожению большого количества разрушений. Об этом событии лампово написано в трилогии Марса от Кима Робинсона. Единственное видео засняла автоматическая камера - проносящийся по экватору огненный шнур от поверхности до атмосферы. Народ поблизости принял ислам.
316796316799316808
#64 #316787
>>316771
ВИРТУАЛЬНЫЕФОТОНЫ
Это всего-лишь модель, антониониоуоус.
#65 #316791
Такой вопрос : Можно ли встать на орбиту внутри чёрной дыры?
316792316935
#66 #316792
>>316791
Нет. Там не то пространство-время к которому мы привыкли и в пределах горизонта событий ты существовать не сможешь, не то что на орбиты выходить
316936
#67 #316796
>>316781

>Я думал, что всем очевидно, что горизонт событий - гроб гроб кладбище пидор и одним куском его не пересечь.


>всем очевидно


Ясно блять. Ну давай, очевидный, поясняй каким хуем горизонт у СМЧД будет "сам пидорасить".
316797
#68 #316797
>>316796
1. Горизонт событий - граница скорость убегания из которой равна скорости света.
2. Все взаимодействия ограничены скоростью света.
Продолжать?
На всякий случай продолжу: Как только часть какой-то материи (твоей ноги, твоего корабля) пересечёт ГС - с той частью не будет никаких взаимодействий. Всё равно что аккуратно на субатомном уровне отрезать часть и удалить. Оно по атому, по кварку переместится из зоны где ещё что-то можно делать, где еще есть взаимодействия друг с другом в зону где взаимодействий с первой зоной нет.
316801316802
#69 #316799
>>316781

>лифт очень брутально намотается на экватор


Уверен, что в случае разрыва какой-либо секции будет большой пиздец, но в том числе уверен, что будет это в целом иначе, нежели описано в очередной нф-писанине.
Ты можешь сколько угодно упираться, доказывая, что твой Робинсон - твердейший твердодел, однако в реальности за анализ такой катастрофы не возьмется ни один разумный инженер-физик. Потому что сейчас даже никто не готов заявить из каких материалов вообще будет строиться, если будет, подобный объект. Пресловутые нанотрубки могут быть получены несколькими разными способами, и в зависимости от способа физические свойства оных будут вполне отличаться.
Так что оставь художественные примеры для /sf.
316803
#70 #316801
>>316797
Атом по кварку отделяется тук-тук-тук-тук.
https://youtu.be/u5OW0iiL9VI
#71 #316802
>>316797

>Продолжать?


Нет, не стоит. Съеби пожалуйста отсюда, ты меня пугаешь своим невежеством и уверенностью в этом невежестве.
316804
#72 #316803
>>316799

>Уверен, что в случае разрыва какой-либо секции будет большой пиздец, но в том числе уверен, что будет это в целом иначе, нежели описано в очередной нф-писанине.


На земле - наверняка да, я забыл упомянуть, что в Марсе говорилось о лифте на Марсе. Логично предположить, что будучи сделанным из нанотрубок он имеет больший запас чем земной и может намотаться.

>Ты можешь сколько угодно упираться, доказывая, что твой Робинсон - твердейший твердодел, однако в реальности за анализ такой катастрофы не возьмется ни один разумный инженер-физик.


Не могу не согласиться.

>Так что оставь художественные примеры для /sf.


Не могу согласиться. Некоторые примеры очень актуальны для спейсача: черную дыру нолана сколько уже обсасывали, Гермес из марсианина, и вообще многие неплохие бибисишные космофильмы основаны на допущениях и домыслах и не являются документалками.
#73 #316804
>>316802
Сиди и помалкивай, если не можешь в диалог.
Два логичных тезиса тебе предложены в посте из которых приводится логичное следствие. Ты не поспорил ни с одним из тезисов и не указал на логическую ошибку в следствии.
316808316809
sage #74 #316808
>>316781

>ГС, который уже сам распидорасит


>всем очевидно, что горизонт событий - гроб гроб кладбище пидор и одним куском его не пересечь.


А мне очевидно, что ты пидор.

>У нас полно радиотелескопов


У кого у нас, блять?

>в пределах горизонта событий ты существовать не сможешь


>>316804

>Сиди и помалкивай, если не можешь в диалог


Как же у меня бомбит от тупого самоуверенного школьника. Ты не привёл ни одного аргумента в пользу своей точки зрения, и ещё кудахтаешь про неумение вести диалог.
Сажи пидору.
316812
#75 #316809
>>316804
Ок, каким образом на основе представлений о горизонте событий ты делаешь вывод о состоянии материи за горизонтом. Где твой аргумент за то, что материя растирается в самую мелкую кашу под горизонтом? Взаимодействия ограничены скоростью света, на горизонте событий скорость убегания равна скорости света. Под горизонтом скорость убегания ее превышает, но как это мешает взаимодействовать взаимодействиям на скорости света? Не будем говорить о том, что для оперирования логикой в столь умозрительных экспериментах нужно иметь очень нихуевый базис знаний в квантмехе и астрофизике, которых нет ни у меня, и тем более ни у тебя. Да и пожалуй вообще вся современная теоретическая физика еще очень шатко работает в этом вопросе.
Но просто теоретически предполагается, что на разрыв материи в труху влияет именно градиент гравитации, который у горизонта событий СМЧД имеет вполне умеренное значение. Никто не делает никаких выводов о том, что происходит с материей под горизонтом, ибо антинаучно. И если бы у тебя был трезвый базис на основе хотя бы научпопа, ты бы тоже хуйни не порол.
316812
#76 #316812
>>316808

>пидор


>блять


>Сажи пидору


>Как же у меня бомбит от тупого самоуверенного школьника.


Не беда, подрастёшь, пубертатный период пройдёт, перестанешь на себя злиться.
А пока поучу тебя вести дискуссию: ты говоришь, что ты не принимаешь на слово, если ты этой информацией не владеешь, я предоставляю тебе её, либо сразу даёшь контраргумент опровергая мои слова, как это положено делать в цивильном месте коим я считаю спейсач.

>>316809

>Где твой аргумент за то, что материя растирается в самую мелкую кашу под горизонтом?


>Под горизонтом скорость убегания ее превышает, но как это мешает взаимодействовать взаимодействиям на скорости света?


Это и является ответом: атомы держатся вместе в молекулах и составляют тела с помощью электромагнитного, слабого и сильного ядерных взаимодействий, которые осуществляются фотонами, W, Z-бозонами и глюонами.
Достаточно уже только остановить перенос хотя бы одного вида этих частиц чтобы материя перестала существовать в привычном виде (распалась, например). А т.к. мы знаем, что скорость убегания выше скорости света, то ни один фотон, бозон или что угодно не достанет из-под ЧД да и в процессе её пересекания материя будет отслаиваться словно на атомарной тёрке: пересекшие атомы никак не будут связаны с непересекшими.
Можно попробовать для интереса придумать умозрительный эксперимент с коробкой у которой посередке выстлано сферическим плоским горизонтом событий в вакууме (ну как-нибудь хитро стырил у СМЧД часть поверхности скрутив пространство-время, для эксперимента это не важно) и в эксперименте можно пренебречь градиентом гравитации и вообще считать что притяжения практически нет. На дне коробки математический конструкт который требует превышения скорости света для того чтобы выбраться обратно. Если абстрагироваться от прочих параметров черных дыр что случится с атомами идеального алмазного стержня (или нанотрубки, или пальца) которые доберутся до горизонта событий?

Естественно, в своем ответе я не учитываю никаких релятивистских эффектов, как и не опираюсь ни на какие неизвестные науке феномены для простоты ответа.
На самом деле объяснить, что будет с человеком пока невозможно, можно только предположить, и мой ответ это один из ответов помимо спагеттификации, поджаривания в джете или диске и прочих аспектах сопутствующих нашим предположениям о ЧД.
А оригинальные вопросы как раз о таком аспекте и спрашивают: что будет если залезть в ЧД.
Мой ответ: залезшая материя перестанет реагировать с незалезшей, ее просто отрежет.
Если у вас есть другой ответ кроме пидор пошёл нахуй - давайте обсудим.
#76 #316812
>>316808

>пидор


>блять


>Сажи пидору


>Как же у меня бомбит от тупого самоуверенного школьника.


Не беда, подрастёшь, пубертатный период пройдёт, перестанешь на себя злиться.
А пока поучу тебя вести дискуссию: ты говоришь, что ты не принимаешь на слово, если ты этой информацией не владеешь, я предоставляю тебе её, либо сразу даёшь контраргумент опровергая мои слова, как это положено делать в цивильном месте коим я считаю спейсач.

>>316809

>Где твой аргумент за то, что материя растирается в самую мелкую кашу под горизонтом?


>Под горизонтом скорость убегания ее превышает, но как это мешает взаимодействовать взаимодействиям на скорости света?


Это и является ответом: атомы держатся вместе в молекулах и составляют тела с помощью электромагнитного, слабого и сильного ядерных взаимодействий, которые осуществляются фотонами, W, Z-бозонами и глюонами.
Достаточно уже только остановить перенос хотя бы одного вида этих частиц чтобы материя перестала существовать в привычном виде (распалась, например). А т.к. мы знаем, что скорость убегания выше скорости света, то ни один фотон, бозон или что угодно не достанет из-под ЧД да и в процессе её пересекания материя будет отслаиваться словно на атомарной тёрке: пересекшие атомы никак не будут связаны с непересекшими.
Можно попробовать для интереса придумать умозрительный эксперимент с коробкой у которой посередке выстлано сферическим плоским горизонтом событий в вакууме (ну как-нибудь хитро стырил у СМЧД часть поверхности скрутив пространство-время, для эксперимента это не важно) и в эксперименте можно пренебречь градиентом гравитации и вообще считать что притяжения практически нет. На дне коробки математический конструкт который требует превышения скорости света для того чтобы выбраться обратно. Если абстрагироваться от прочих параметров черных дыр что случится с атомами идеального алмазного стержня (или нанотрубки, или пальца) которые доберутся до горизонта событий?

Естественно, в своем ответе я не учитываю никаких релятивистских эффектов, как и не опираюсь ни на какие неизвестные науке феномены для простоты ответа.
На самом деле объяснить, что будет с человеком пока невозможно, можно только предположить, и мой ответ это один из ответов помимо спагеттификации, поджаривания в джете или диске и прочих аспектах сопутствующих нашим предположениям о ЧД.
А оригинальные вопросы как раз о таком аспекте и спрашивают: что будет если залезть в ЧД.
Мой ответ: залезшая материя перестанет реагировать с незалезшей, ее просто отрежет.
Если у вас есть другой ответ кроме пидор пошёл нахуй - давайте обсудим.
316813316814317245
#77 #316813
>>316812
Ты упорен, ты не хочешь внимать, видимо тебе недостаточно авторитета местных "дурачков-воннаби астрофизиков". Ну, как знаешь, на мой взгляд ты называешь очевидным совершенно не очевидное, а для меня это болезненный триггер в этом разделе. Однако я уважаю твою манеру разговора, размерено и конструктивно, хоть и впустую, диалог завершаю. Последнее слово - за тобой, если пожелаешь.
316814316899316926
#78 #316814
>>316812
>>316813
Два дебила - это сила.
Вернитесь в /б оба.
316816
#79 #316816
>>316814
Да это один, базарю.
#80 #316822
Ответе на мои ответы, пожалуйста.
>>316622
>>316722
316834316897
#81 #316834
>>316822
Нет.
#82 #316837
>>316722

>Зачем


У неё спроси. Избавь нас от своей тупости.
#83 #316897
>>316822
>>316622
Космонавтика - чтобы летать над Землёй, всё видеть изучать там эксперименты вот это всё.
#84 #316899
>>316813

>Последнее слово - за тобой, если пожелаешь.


Спец олимпиада во всей красе. За мной последнее слово
316926
#85 #316900
Можно ли терраформировать черную дыру?
316944
#86 #316903
>>316443 (OP)
У меня возник наверное очень тупой и очевидный вопрос
Но все мы привыкли к тому, что все имеет свой конец. Мы знаем пределы своей квартиры, своего района, своей страны. Но как вообще понять устройство вселенной и всего пространства. Оно бесконечно? Это просто не укладывается в голове. ведь не бывает ничего бесконечного. Даже если есть какой-то предел, то что-то ведь должно быть и за пределом. По моему человеческий мозг в принципе не запрограммирован на понимание бесконечности или конечности. Все это крайне непонятно. И вопрос такой. А ученые вообще что-нибудь по этому поводу иcследовали?
316906317638
#87 #316906
>>316903
Всегда такое читаю и думаю хуйня какая-то. Все привыкли что по всему миру не поездишь и всех баб не перетрахаешь. Все знакомы с условно бесконечными ресурсами.
316907
#88 #316907
>>316906
Да, но теоретически ты можешь все это охватить. На всех людей из данные, на каждый уголок земли есть данные
316918
#89 #316912
>>316719
А кому еще, собственно? В стране никто больше не имеет опыта постройки АМС. ИСС? Ну они могут ИСЗ делать, но АМС в любом виде это совершенно другое. Да и опыт лавке нужен, хотя бы так.
316917
#90 #316917
>>316912
Самим надо было делать. Европейские зонды хотя бы не сыпятся из-за БЦВМ, спроектированной студентом.
Перспектива разъебать марсоход, это, конечно, лучше, да.
316948
#91 #316918
>>316907

>Да, но теоретически ты можешь все это охватить


Типа по текущим представлениям тебе не удастся путешествовать по вселенной со скоростью, превышающей предельную. И всякая ерунда, вроде света, уже какое-то время с того момента, как все завертелось, летит по всем направлениям. Так что сколько бы ты ни путешествовал, "край" вселенной, либо какую-то другую область, обозначающую "конец", все равно не догонишь. Я так себе это понимаю.
#92 #316926
>>316813
>>316899
Что за дроч на последнее слово?
Оно не всегда а точнее почти никогда не решающее.
#93 #316929
Почему тайконавта так плющит, что он не может на одном месте висеть?
https://www.youtube.com/watch?v=zXA2jhTPiC4
67 Кб, 504x373
#94 #316935
>>316690

>Можно ли встать на орбиту внутри чёрной дыры?


>>316791

>Нет. Там не то пространство-время к которому мы привыкли


Структура пространство-времени там действительно отличается, и очень сильно, но из этого вовсе не следует такой категорический вывод. Правильный ответ: можно, но не внутри любой. Внутри шварцшильдовской ЧД все мировые линии заканчиваются на сингулярности, так что там без вариантов. С другой стороны, почти все реальные ЧД вращаются, а внутри керровской ЧД под горизонтом событий есть второй внутренний горизонт — горизонт Коши, под которым вполне возможно существование стабильных орбит. Правда, часть из них имеют весьма наркоманский вид из-за дикой прецессии, вызванной близостью сингулярности.
316936
#95 #316936
#96 #316943
https://www.youtube.com/watch?v=szN2Clyzz7Y

зачем они выбрасывают лишние железяки за борт? американцев-водолазов прикармливают?
#97 #316944
>>316900
Стоит попробовать
20 Кб, 512x361
#98 #316948
>>316917
Самому это кому? Одному из предприятий ЕКА, в смысле?

>Европейские зонды хотя бы не сыпятся из-за БЦВМ, спроектированной студентом.


Например EDM, да? У еврокомпаний тоже весьма хуевая история с лендерами (даже с успешным Гюйгенсом случилась организационная проблема с нестыковкой спецификаций и передачей команд с Кассини, в результате которой проебали и героически спасали эксперимент со скоростью ветра), и их тоже разрабатывают в том числе и стажеры, представь себе. Как и везде. Кроме того, в составе ЕКА нет компаний с опытом построения цельных систем хотя бы уровня лавки, т.к. там в основном подход "всем раздать понемногу, чтобы не было обидно" (как в случае с Ариан-6 и Розеттой, которую клепали с миру по нитке). К примеру TAS, сделавшая EDM, это эквивалент ИСС скорее. Вся миссия совместная, и в ней какую часть ни отдай - любая окажется критической, в ней мало резерва.

>Перспектива разъебать марсоход, это, конечно, лучше, да.


Перестань мыслить стереотипами и шаблонами, и включи здравый смысл. То что кто-то в спешке халатно отнесся к разработке одного аппарата, совсем не обязательно означает, что будет бесконечно проебывать все остальные и что ему нельзя давать никакой работы никогда и надо вечно гореть в адском пламени и чтобы всегда как в первый раз. Уууу, негодяи, всё распилили! Платформу Навигатор с тем самым БКУ, зафейлившим на ФГ, они таки доделали в тех же Электро-Л, например - но про это уже мало кто вспомнил за мемесами и квадратиками.
#99 #316951
Анон, поясни за климат на планете-океане.
Берём, короче Землю со всем теми же параметрами расположения, ресурсов, луны, температуры и т.д., но тяпляпаем сушу так, что в итоге планета Земля становится планетой Водой со множеством далеко или не очень расположенных друг к другу островов. Размеры островов от самых мелких до, британских или даже Японии. Расположение пускай будет в тех местах, где у нас сейчас Евразия, Африка, Америки и Австралия. Острова Океании или острова, не входящие в указанные регионы пост-суши тонут.
Собственно, вопрос по климату. Изменится ли скорость ветров, влажность, температура и всё такое на планете?
Как это скажется на жителях островов? На флоре и фауне островов?
Подозреваю куда более высокую влажность, более быстрые ветра, куда более высоки волны (особенно подле морей и океанов на рубеже островного сегмента атласа и сегмента бескрайних вод). Не знаю, как это повлияет на Гольфстрим и на ледники.
316952
#100 #316952
>>316951
а подводные горы есть?
316955
#101 #316955
>>316952
Ну да. Местами есть. Вред ли же такая планета, похожая на булаву, где вся поверхность планеты - равномерное глубоке дно под толщей воды, и только шипы на такой булове - высоко торчащи егоры над дном, а на поверхности они превращаются в клочки суши.
41 Кб, 711x711
#102 #316957
На какую глубину нужно погрузиться человеку, что бы разница в давлении между внешней средой и внутренней (внутри тела или скафандра) была равно разнице давлений при выходе человека в открытый космос?
316958317023
#103 #316958
>>316957
человек-краб у нас жёсткий?
317008
112 Кб, 320x320
#104 #316959
Объясните, откуда к каждому ютубному видосу про космос столько плоскоземельцев? Нет никакого космоса! Причем, заходишь в профиль - не тролль, реальный ебанавт.
316961316964316965
#105 #316961
>>316959
ну хорошо. а как ты без ссылок на научные пруфы сам докажешь обратное?
вот докажи мне
316962
#106 #316962
>>316961
Хм... Вот эбаут линия горизонта?
316963
#107 #316963
>>316962
Изогнутость земли видно только на большой высоте, недоступной для тебя.

На Земле ты видишь только лишь что если смотреть вдаль то не видно далеко. Но может это из за плотности воздуха?

Кроме того лёгкая изогнуость ещё не знает круглость. Может земля легкоизогнутая плоскость а не шар.
173 Кб, 1121x636
#108 #316964
>>316959
Как будто верить в Бога чем то лучше.
316973
106 Кб, 1251x883
#109 #316965
>>316959
Они умело используют систему тегов добавляемых к видео, добавляя к своим видео теги типа "Space", "Earth", "Astronauts". В результате их видео оказываются в рекомендуемых у тысяч людей. Люди, которые хоть раз даже по ошибке включили видео плоскоземлян, продолжают получать в рекомендуемых их видео.
Они примерно год назад на ютуб перебрались из глухих форумов, где ошивалось 3.5 инвалида-аутиста, а сейчас у них видео больше, чем у науч-поп мейкеров. Качество их видео конечно говно, но с тегами работает именно тактика количества.
#110 #316973
>>316964
Типнул федору.
#111 #317003
На сколько сильна рашка в космосе сейчас ? Че там по спутникам, миссиям. Или в космосе мы только как поставители двигателей ?
И на сколько движки влияют ? Вот откажемся мы их продавать и что тогда ? Не будет запусков или просто аналоги дорогие ?
317024
1110 Кб, 1068x598
#112 #317007
>>316443 (OP)
Что это за скороварка? Примерно где-то на модуле Гармонь, если не ошибаюсь.
317013
#113 #317008
>>316958

> человек-краб у нас жёсткий?


Молодой человек, это не /bb/, вы разделом ошиблись
626 Кб, 3032x2008
#114 #317013
>>317007
Это ты про захват Канадарма2?
#115 #317018
Аноны, я вот нихуя не понимаю, что значит отрицательная энергия или антиматерия, или антивещество. У меня в голове не укладывается. Можете дать статейку или разжевать дауну - что это такое.
317030317049
#116 #317023
>>316957
Погуглил за тебя - 10.333 м. только в обратную сторону, естественно
317025
#117 #317024
>>317003

>Че там по спутникам, миссиям.


МКС, регулярно к станции летают пилотируемые и грузовые корабли. Новые модули делают, да всё как-то долго не доделывают.
Есть ГЛОНАСС; спутников ДЗЗ прилично крутится вокруг планеты.
На орбите Спектр-Р; пара обсерваторий в производстве, ещё пара разрабатывается - международные проекты. По АМС планов громадьё, но все на среднюю-дальнюю перспективы. Ближайшая - посадочная платформа ЭкзоМарс-2020; несколько (тут не уверен) Лун в производстве.

>Не будет запусков или просто аналоги дорогие ?


Двигатели российского производства стоят на РН Atlas V (целое семейство, используется в основном для запуска военных спутников связи, навигации и разведки и исследовательских КА, в том числе АМС) и Antares. Двигателей-аналогов в США нет. Но есть РН сходной грузоподъёмности - семейство Delta IV и Falcon 9. Первые неебически дорогие и используются только военными, а второй уже забрал себе почти все коммерческие пуски среди американских РН. Для замены Atlas V разрабатывается РН Vulcan.
Без российских двигателей по пизде пойдут грузовой корабль Cygnus и пилотируемый Starliner, так как первый запускают на Antares или Atlas V 401, а второй будут запускать с помощью Atlas V 422. Но в таком случае все их функции полностью перейдут на Дрэгоны от SpaceX.
#118 #317025
>>317023
Спасибо.
Значит, осваивать и оттачивать действия в открытом космосе в экстренных случаях нужно где-то в маринской впадине?
317026
#119 #317026
>>317025
Ты точку пропустил.
317029
#120 #317029
>>317026
Где?
317031
#121 #317030
>>317018
Кароч есть уравнения гравитационные от Эйнштейна. По ним рассчитывают че куда полетит. Конечный результат зависит от массы тел.
Так вот, на самом деле эти уравнения нихуя не работают, поэтому и придумали, что есть некая материя, которую не видно и нельзя обнаружить =))))
317049317162
#122 #317031
>>317029
Между 0 и 3
318704
#123 #317048
https://ru.wikipedia.org/wiki/Предел_Грайзена_—_Зацепина_—_Кузьмина
Очередной пример непригодности теории относительности. А тут питухи спорят о том, что же будет, если войти в черную дыру. Вы хотя бы в вики зайдите, почитайте что-нибудь кроме страницы про черные дыры.
317050
#124 #317049
>>317018
Отрицательная энергия довольно забавная хуйня. Вакуум - не есть состояние абсолютного покоя, или состояния нулевой энергии, это самое низкое энергетическое состояние, но тем не менее вакуум постоянно рождает частицы, которые тут же аннигилируются. То есть из вакуума на время берется энергия и тут же в этот вакуум возвращается. Вот есть две незаряженные пластины в вакууме, они далеко друг от друга, и все как бы окей, но если их приблизить очень близко друг к другу. То в определенный момент получится, что смогут рождаться фотоны с одной длиной волны, так как другие длинны влн уже не укладываются между пластинами, получается, что рождение фотонов снаружи пластин гораздо больше, чем между ними, и пластины слипаются. Вот эта хуйня называется отрицательной плотностью энергии, сам эффект носит название эффект Казимира.

(Есть еще гипотетическая экзотическая материя, которую, в отличии эффекта Казимира никто никогда не наблюдал - это просто продукт рассуждений людей с большим мозгом и кучей свободного времени. Например, что гипотетически может существовать отрицательная масса, если ты толкнешь шар с отрицательной массой, он, как ни странно полетит тебе в ебало, а не от тебя. Но это уже очень все вилами писано.)

>антиматерия, или антивещество


антиматерия = антивещество
Антивещество состоит из античастиц
Античастица -это тоже самое, что и частица, то есть ее двойник, с точно такой же массой, с таким же спином, но при этом имеет противоположный по знаку заряд и лептонное число.
Например античастица электрона - позитрон, имеет как электрон точно такую же массу, такой же спин 1/2, но заряд не -1, как у электрона, а +1, и лептонное число -1, а не +1, как у электрона.

>>317030
Кефир, уходи.
317052
#125 #317050
>>317048
Ты бы сам по своим ссылкам ходил, петух. И читал те статьи, что ты сюда носишь до конца.
317052
#126 #317052
>>317049
Ты хотел сказать "кяфир"? Шатается вера твоя?

>>317050
А ты-то прочитал?

>наблюдая только 13 событий с энергией выше порога, при ожидаемых 43 без подавления


>вместо 30 событий, необходимых, чтобы подтвердить результаты AGASA, наблюдались только 2 события.


13 из 43ех и 2 из 30ти пробивают предел.

Помолиться не забудь сегодня на усики Эйнштейна.
317054
#127 #317054
>>317052
Сука, до чего ты тупой блядь. Сам несет сюда хуйню, что ГЗК парадокс не подтвержден и радуется как ребенок.

На вот, покушай.
https://arxiv.org/abs/0704.2463
317060
#128 #317060
>>317054
Последняя надежда на то, что кяфир не знает английского? Завтра почитаю.
317092
#129 #317064
Бывают ли звезды с 10+% металличностью? Каковы особенности их эволюции? Я слышал, это при равной массе даёт большую температуру/светимость.
2241 Кб, Webm
#130 #317092
>>317060

>Последняя надежда


Нет никакой надежды в отношении дебила, который не отличает отрицательную энергию, антиматерию и темную энергию с темной материей.
Чего ждать от дебила, который в открытую тащит в тред ссылки по которым же сам ест говно.
О чем вообще разговаривать?
317142
59 Кб, 860x403
#131 #317095
Что за шумиха с ЭмДрайв?
Как должен работать по-идее?
Как он работает как бы по факту?
В чём хайп?
Какие законы физики он, вроде как, нарушает?
317097
#132 #317097
>>317095
Иди в тред эмдрайва, че ты как этот то.
317099
#133 #317099
>>317097
Там хуй по трреду размазан, а в ТТВ космосача мне смогут коротко и ясно ответить
317102
#134 #317102
>>317099

> что за шумиха


Микроволновка двигает себя больше, чем должна.

> как по идее


Ну есть конечно давление фотонов, но оно слабее, и с направленностью проблемы.

> как по факту


Проверяли в вакууме, но на Земле. Часть возможных источников ошибки отсеяли, часть остаётся.

> в чём хайп


Берёшь не оче большой спутник с достаточно мощными солнечными батареями и выбрасываешь на НОО. Дальше он сам доползает, куда надо (куда — тоже ещё вопрос, может только в магнитном поле Земли и работает) не надо ебаться с движочками и гидразином, топливо не кончится, station keeping простейший.
703 Кб, 2612x2412
#135 #317139
Как думаете, сможет ли текущее поколение лицезреть свободные межпланетные полёты? То есть полёт на Плутон был бы такой же обыденностью как и полёт в другую страну.
#136 #317140
>>317139
А тебе зачем?
317143
270 Кб, 1024x576
#137 #317142
#138 #317143
>>317140
Считал бы это самым большим достижением человечества, да и космос представляет для меня огромный интерес.
317144317161
#139 #317144
>>317143

>да и космос представляет для меня огромный интерес


Настолько огромный, что ты даже зашел на двачь в тематику? Да еще и пост написал? =))
317154
#140 #317145
>>317139

> полёт на Плутон


> текущее поколение


Если бы те кто родились в десятых, единичный флаговтыкательный полет на Марс при жизни увидели, и то было заебись.
317146
#141 #317146
>>317145
Воще-то в натуре должны в 30ых полететь уже с флагом.
317147317153
#142 #317147
>>317146
Я прослежу, если что.
#143 #317153
>>317146
Ага, а до этого собирались в двадцатых. И до этого собирались. И до того...
#144 #317154
>>317144
Ну я же не ботан какой-то.
317155
#145 #317155
>>317154

>я же не ботан


Докажи или ботан.
317158
#146 #317158
>>317155
Как ты уже сказал, я "даже зашел на двачь в тематику? Да еще и пост написал".
317159
0 Кб, 512x512
#147 #317159
>>317158

>эти ботанские отмазки


Ясно.
#148 #317161
>>317143
Ну слетаешь ты на Плутон, ну ебало разинешь, фотачке, селфи хуелфи, ну шлюху нептунианскую выебешь. Дальше то что?
317178
#149 #317162
>>317030

> придумали


Пора уже мочеру пидорнуть отщепенцев с сцайфай на пару с их мертвым тредом.
#150 #317163
Чо там у falcon heavy? Когда НА АЛЬДЕБАРАН МАРС?
317169
#151 #317169
#152 #317178
>>317161

>шлюху нептунианскую выебешь


Тогда я потеряю девственность как и всегда хотел, заебись.

>Ну слетаешь ты на Плутон


Такого люди не могли себе позволить никогда, а тут я такой делаю селфи, ну как не хотеть такого?
#153 #317212
>>317139

>полёт на Плутон


Хули там делать, МиГо кормить?
317226
#154 #317226
>>317212
Не важно что, главное что появится возможность вступить на другую планету.
317235
#155 #317235
>>317226

>вступить


Вступать ты в партию будешь.
И вообще Марс ближе. И Плутон - сам знаешь, что
#156 #317245
>>316812
Двачую этого. Очень интересная гипотеза.
#157 #317278
Атмосфера Титана может поддерживать горение?
317280
#158 #317280
>>317278
Нет, кислорода нема.
317281
#159 #317281
>>317280
Так ведь не только в кислороде горение возможно.
317283
#160 #317283
>>317281
Там 95-98 % азот, остальное метан, этан в чем ты там жечь собрался?
23 Кб, 540x176
#161 #317286
>>317139

>межпланетные


>Плутон


Тонко.

Вопрос: как учёные добиваются финансирования исследований космоса? Дорого ведь, и не приносит такой очевидной пользы как медицина/ядерная физика/генетика.
Единственная выгода которая мне приходит в голову - спутники (коммуникационные, разведывательные, военные).
317287317292
#162 #317287
>>317286
Те же горизонты какой-то шейх спонсировал.
#163 #317291
Если когда-нибудь люди изобретут сверхсветовой двигатель, телепортатор в любую точку вселенной etc.
Можно ли будет поймать Voyager-1, запеленговать его как-то и т.д.? Он же совсем небольшой.
317293
#164 #317292
>>317286
Госфинансирование, как и любые другие фундаментальные исследования. Коммерческий космос к исследовательским АМС никаким боком.
#165 #317293
>>317291
После того как заглохнет - напрямую нет. Можно будет по траектории определить его позицию с некоторой точностью. (если модель межзвездного пространства будет верна и он не будет лететь быстрей/медленней чем предсказывалось)
#166 #317294
Если все фотоны и прочие безмассовые уйдут за космологический горизонт, а внутри ,к примеру, будет только 1 сгусток материи, части которой отличимы друг от друга только давлением, это называют энтропией?
317301
#167 #317301
>>317294
Энтропия сама по себе это просто неупорядоченность. Она по жизни существует и возрастает.
А ты описываешь некое подобие космологических сценариев смерти вселенной. Даже какое-то совмещение их. Уход чего-либо за горизонт это большой разрыв, неотличимость материи это тепловая смерть.
317347
245 Кб, 800x954
#168 #317342
Анон, пикрелейт - фотомонтаж или реальное фото? От космической тематики далековат но охуел бы, будь это правдой. Гугл выдает ссылки на всякие пикабу и проч юмор, где по делу ничего не найти.
317343317401
#169 #317343
>>317342
Фейк, инфа соточка.
#170 #317347
>>317301
Это просто арт.
317368
#171 #317368
>>317347
Слишком ленивый арт, слепленный из нескольких фотографий и обработанный в фотошопе дабы скрыть швы.
312 Кб, 600x600
#172 #317369
Если предположить, что вселенная не расширяется с бешеной скоростью и галактики не стремятся слится в одну большую, а звёзды горят стабильно с одной температурой вечно, то что произойдёт с жизнью на отдельно взятой планете, которая никогда не сойдёт с орбиты и не будет уничтожена извне или антропогенным фактором?
317371
#173 #317371
>>317369
Она умрёт.
317400
#174 #317400
>>317371
Лол. На бесконечности времени она потом оживает, а иисус верхом на динозавре расстреливает зомби нацистов из рпг-7.
Я имею ввиду, ввиду чего?
317744
#175 #317401
>>317342
Настоящий. Фото со съемок новой части Звездных Войн в космосе.
7084 Кб, Webm
#176 #317403
Срочно нужно мнение экспертов. Это луна, или что-то что пыталось ей прикинуться? Город Кливленд. Не слишком ли она быстро летит?
317405
#177 #317405
>>317403
Восход Луны никогда не видел, или что?
317406317407
#178 #317406
>>317405
За 15 минут поднялась над городом, это норм?
317409
319 Кб, 512x512
#179 #317407
7960 Кб, Webm
#180 #317408
#181 #317409
>>317406
Это таймлапс, посмотри с какой скоростью там мелькают фары автомобилей.
317410
#182 #317410
>>317409
Обычное видео с вебкамеры, вот https://dropfile.to/nvuKA93
317411317414
#183 #317411
Ну что, вы согласны, что это что-то, что пыталось притвориться луной? Но это НЛО?>>317410
317412
#184 #317412
>>317411
Я, да.
160 Кб, 1072x564
#185 #317414
>>317410

>Perched high on a cliff overlooking Lake Erie, enjoy a live streaming look at the Cleveland city skyline from this HD webcam. Thanks to the Pier W Restaurant, viewers can take in the beautiful sights and sunsets from this waterside locale.


Луна над Кливлендом поднимается примерно на 5° за полчаса и удаленность этой камеры, расположенной в ебеневом ресторане Питера W/ от собственно места съемки, все вполне соответствует. Ибо небоскребы кажутся огромными, но они в ебенях, так что при длинном фокусе планы накладываются друг на друга. Кажется, что луна прошла полнеба, на деле она там на 2.5 градуса залезла и съебала в облака.
317415
#186 #317415
>>317414
Ты просто не веришь в нло
317416
#187 #317416
>>317415

>Мам, я траллю спейсач!

282 Кб, 1920x1080
#188 #317419
Какой радиационный фон/воздействия нужно выдерживать телу человека на околоземной орбите/солнечной системе/Марсе/на орбите Юпитера?
Как можно решить проблему вывода из (органического) тела радионуклидов? Проще ли экранировать тело/спесшип от радиации, или будет перевес? Если не экранировать и решить проблему с выводом радиации, то насколько стабильна будет нервная система? Придётся ли в условиях постоянной радиационной нагрузки заменять части ЦП человека и делать бэкапы десятками? Сколько будут весить «бэкапы»? Если делать человека из органики, будет ли он лучше неорганического тела (с неорганическим мозгом, соответственно)? Возможно ли создать неорганический мозг, или же гибкие и стабильные органические механизмы будут иметь преимущество в условиях радиации?

немножко тупых вопросов перед сном
317426
#189 #317421
Где-то услышал, что ускоряя короткоживущие частицы до околосветовых скоростей можно добиться, что они будут существовать дольше для нас, так как для них время искажается во время их движения. Это так? Можно какой-нибудь супер-дупер тяжелый атом создать, разогнать его в пушке до околосветовых скоростей и ебашить им как из пушки?
317423317433
#190 #317423
>>317421

> Где-то услышал, что ускоряя короткоживущие частицы до околосветовых скоростей можно добиться, что они будут существовать дольше для нас, так как для них время искажается во время их движения. Это так?


это так
#191 #317426
>>317419

> Какой радиационный фон/воздействия нужно выдерживать телу человека на околоземной орбите/солнечной системе/Марсе/на орбите Юпитера?


частицы отклоняются в магнитосфере, накапливаются в радиационных поясах
космический фон рассеивается с приближением к солнцу, фон от солнца рассеивается с квадратом расстояния от него
всё сложно, циферки тебе ничего не скажут

> Как можно решить проблему вывода из (органического) тела радионуклидов?


Проблема не в нуклидах а в ионизации.

> Проще ли экранировать тело/спесшип от радиации, или будет перевес?


Смотря чем экранировать, да и не защититься от всего спектра.

По остальным вопросам в сайфач.
#192 #317433
>>317421

>Можно какой-нибудь супер-дупер тяжелый атом создать, разогнать его в пушке до околосветовых скоростей и ебашить им как из пушки?


Можно, но эффект получится не совсем как из пушки.
#193 #317465
Если много кидать в черную дыру, получится ли так, что чд набрав критическую массу опять разуплотнится во что-то видимое, типа нейронной звезды ну или хуй знает. А если нет, то почему нет?
317466317467
#194 #317466
>>317465
Замедление времени и типа "никогда не упадет за горизонт" в расчет не берем.
#195 #317467
>>317465
Нет у ЧД критической массы, можешь звезды миллиардами накидывать. Сожрет и попросит еще.
#196 #317468
Вопрос из математики бесконечностей.
Есть 3х мерная сетка, 2 объекта, 6 степеней свободы, каждый шаг отклоняет обе частицы на рандомную клетку.
Какова вероятность, что 2 объекта встретятся?
317485
#197 #317478
Как сломать орбитальную механику?
#198 #317481
Как сделать скорость света отрицательной в практике?
317487317488
#199 #317482
Посоны, вы заметили, что с каждым днём время летит все быстрее и быстрее, недавно был 2012 год, а сейчас уже 2017 почти. Наша галактика все дальше удаляется от большого аттрактора и вселенная расширяется все быстрей. А что, если с каждым годом время жизни будет восприниматься нашим мозгом, как нынешняя минута? Что, если в голове человека есть механизм восприятия времени и он зависит от скорости расширения вселенной? и да, я гуманитарий
317483
#200 #317483
>>317482
У меня время наоборот начало течь медленнее
#201 #317484
Из чего делают ракеты? Железо? Титан? Алюминий?
317486
#202 #317485
>>317468
-> /sci/
#203 #317486
>>317484

> люминий


по большей части
#204 #317487
>>317481
запускать свет в противоположную сторону
#205 #317488
>>317481
Скорость - векторная величина
#206 #317489
Скалярный пенис, посоны.
317491
#207 #317491
>>317489
векторный же, есть конец
317494
#208 #317494
>>317491

>векторный


>патамушта есть конец


Вот ты ебанулся что ли? Может у тебя и числовая прямая - вектор?
317495
#209 #317495
>>317494

> числовая прямая


> конец


пиздос
317496
#210 #317496
>>317495
Промежутки не проходили в школе? А множества?
317497
#211 #317497
>>317496

> числовая прямая


> Промежутки


манёвр защитан, хватит уже про фаллосы
317506
#212 #317506
>>317497

>хватит уже про фаллосы


Ты же в разделе про ракеты, чо ты.
312 Кб, 720x338
#213 #317529
Может ли быть большая планета типа Юпитера или Сатурна БЕЗ магнитного поля?
В приницпе-то обитаемая планета-луна могла уютно прижиться при таком гиганте, не будь такой архипиздической адиации из-за полей.
И ещё, а может ли быть планета - точнее, при каких условия и возможна ли вообще обитаемая планета, представляющая из себя два обитаемых полюса с водой и всяким таким, в то время как через экватор проходит безлюдная выветренная и выжженная пустыня. скажем по 2000км к северу и югу от экватора?
И наконец самый тупой вопрос: возможна ли планета в зоне обитаемости с частичной атмосферой? Например - на плюсах есть и густая, а на экваторе - нет, или есть, но пренебрижимо слабая и рязряженая
317531317548317553
#214 #317530
Успеют ли людишки залететь в большое красное пятно и снять в 4к хд до его распада?
317531317532317540
#215 #317531
>>317529
Теоретически может обладать слабым магнитным полем, практически нет.
Что касаемо пригодных для жизни планет то их только 1 шт., и то временно. На любой другой, если только у нас не завалялось артефакта +1010200 удачи будет перекос в каком-то параметре и нам требуется допил тераформингом или в идеале купол.
Атмосфера стелится ровно и если теоретически на экваторе будет горный массив то нам будет слабая атмосфера. Но только если это астероид, планеты слишком жидки для этого.
>>317530
Зачем? Он ведь только снаружи такой красивый. Внутри ведь видимость низкая. Всё равно что говорить про сьёмку мутной воды.
317560
#216 #317532
>>317530
Большое Красное Пятно в 9 раз больше Земли, так что внутри него нихуя никакого Красного Пятна просто не видно. Это все равно что рассматривать всю Москву с балкона пятиэтажки где-нибудь в ЮЗАО.
#217 #317540
>>317530

>до его распада


Ему недолго осталось?
#218 #317548
>>317529
лел, у Юпа есть Каллисто
она вне радивционного пояса, а приливный захват гарантирует отсутствие на внешней стороне выбросов с самого Юпа
#219 #317553
>>317529

> Может ли быть большая планета типа Юпитера или Сатурна БЕЗ магнитного поля?


Вообще без магнитосферы - нет, ибо при больших давлениях обязательно образуется всякая экзотическая хуита, которая проводит ток и создает сильное магнитное поле при вращении. Со слабой магнитосферой - вполне, например Сатурн намного слабее в этом плане.

>не будь такой архипиздической адиации из-за полей.


Тащемта как указали выше - спутники могут летать и над ними. Под ними уже нельзя, т.к. это будет ниже предела Роша (например Метис и Андрастея расположены прямо во внутреннем поясе Юпа, при этом находясь под пределом для жидких тел)
317560
1711 Кб, 400x225
#220 #317560
>>317553
>>317531
А может ли быть при слабом магнитным полем газовом гиганте спутник с оче сильным полем?
317562
#221 #317561
Поясните, как формируется угловой момент вращения протопланеты? Есть псевдонаучный текст, вот этот:

the capture of material from orbits which were in general more circular than its own would tend to give a spin to the forming world, since objects from outside its position at any instant would have a lower velocity than those from farther in

пошел в лингвач, там перевели так

Чем дальше объект от планеты, вокруг которой кружит, тем меньше его скорость

Но если я правильно помню школьный курс, все должно быть наоборот? Обьясните, короч.
317562317566
#222 #317562
>>317560
Не вижу ограничений, вроде может

>>317561
Нет, все верно, дальше от центра - меньше скорость.
317563
#223 #317563
>>317562
как тгда понять эту фразу?

На стадии, когда его "атмосфера" распространялась в космос возможно на миллионы миль, материал, захваченный с орбит более округлых, чем его собственная, придавал вращение формирующейся планете, поскольку чем дальше объект от планеты, вокруг которой кружит, тем меньше его скорость.
317567
#224 #317566
>>317561

>Но если я правильно помню школьный курс


Не правильно. Есть четкая зависимость от расстояния между телами.

https://ru.wikipedia.org/wiki/Орбитальная_скорость
#225 #317567
>>317563

>чем дальше объект от планеты, вокруг которой кружит, тем меньше его скорость.



Ну, все правильно.
#226 #317569
Там еще есть дебильный кусок про планетарные кольца, точнее щели между ними. Использованы такие обозначения quarterperiod, third-period, half-period ring gaps что переводится как четвертьпериодичная щель, третьпериодичная и полупериодичная. И даются расстояния от них до поверхности, полупериодичная самая далекая. Что это должно значить, я даже представить не могу.
317571317588
#227 #317570
Какие оптические телескопы самые мощные?
317572
#228 #317571
>>317569
Да, перед этим идет про спутник планеты-гиганта: внутренняя луна значит висит на таком то расстоянии (как и треть-периодичная щель) что дает ей период в 2 часа 8 мин. Может, щели относятся к этому периоду?
317588
#229 #317572
>>317570
Хаббл и ВЛТ
81 Кб, 1000x695
#230 #317573
Тут военным предлагают РЭБ в космос вывести. Это как вообще идея реальная? Можно заглушить GPS из космоса? Тогда спутники не получая сигналов сами начнут падать?
317590
7385 Кб, Webm
#231 #317574
Все же тут играли в КСП?
Вот есть в игре Минмус, вроде как 60км диаметром, состоит, видимо из газопылевых пород, так как находясь в той же зоне, что и Кербин, имеет лёд какой-то из жидкости.
Так вот вопрос: если мы возмём такой Минмус и поместим его в на орбиту Земли, как это повлияет на систему Земля-Луна?
То есть, расстояния, куда его можно посместить от балды беру от 1/3 расстояния до Луны до 2 расстояний.
Будет ли Минмус в орбитальном резонансе с Луной? Если он будет близок к Земле, вызовет ли это какие-нибудь возмущения? Он же, по идее совсем лёгкий, и объём держится чить ли не на водяном паре и пыли.
317584
#232 #317575
Привет, такой вопрос: может ли оказаться что всё что мы видим, вся реальность/материя и т.д. всё это одна-единственная частица? это конечно похоже на бред наркомана но попробую объяснить, на самом деле я просто посмотрел пару видосов на тему квантовой физики и там говорилось о телепортации и возможности одной единственной частицы одновременно находиться в нескольких местах, то есть по сути это не телепортация как таковая. Так вот вроде как на квантовом уровне понятие времени искажается, то есть время как то зависит от того кто это самое время воспринимает, и вообще его восприятие чисто субъективно и зависит от параметров наблюдателя, то есть время на самом деле имеет привязку к пространству и не может существовать вне его или на бесконечно малых величинах, например в сингулярности чёрной дыры оно равно нулю, так вот если взять бесконечно малый объект например фотон и гипотетически изменять его размер то шкала времени будет квантоваться пропорцинально этому самому изменению его размера, так например если взять какойнибудь объект длинной в 10 световых секунд и толкнуть его с одного конца в другой то на другом конце движение произойдёт только спустя эти 10 секунд, это всё о привязке времени к пространству. но мысль сообщения вот в чём, если настолько малые частицы как кванты неподвластны времени, что если один-единственный квант может находиться в один и тот же миг во всех частях вселенной при этом в разных состояниях и формах, порождая этим самым собственно все вещества и материю галактики биологию химию нас с вами и вообще всё, всё это одна-единственная частица существующая вне привязки ко времени в силу своей бесконечно малой величины. и что если биологическая жизнь и наше сознание, то что мы с вами изучаем вселенную, реальность окружающую нас, ну и вообще живём и наблюдаем/ощущаем наш мир, может на самом деле это просто попытки этой частицы познать саму себя и когда мы думаем что познаём мир на самом деле это она через нас познаёт свою природу. ну вот, такая наркоманская теория:D
#232 #317575
Привет, такой вопрос: может ли оказаться что всё что мы видим, вся реальность/материя и т.д. всё это одна-единственная частица? это конечно похоже на бред наркомана но попробую объяснить, на самом деле я просто посмотрел пару видосов на тему квантовой физики и там говорилось о телепортации и возможности одной единственной частицы одновременно находиться в нескольких местах, то есть по сути это не телепортация как таковая. Так вот вроде как на квантовом уровне понятие времени искажается, то есть время как то зависит от того кто это самое время воспринимает, и вообще его восприятие чисто субъективно и зависит от параметров наблюдателя, то есть время на самом деле имеет привязку к пространству и не может существовать вне его или на бесконечно малых величинах, например в сингулярности чёрной дыры оно равно нулю, так вот если взять бесконечно малый объект например фотон и гипотетически изменять его размер то шкала времени будет квантоваться пропорцинально этому самому изменению его размера, так например если взять какойнибудь объект длинной в 10 световых секунд и толкнуть его с одного конца в другой то на другом конце движение произойдёт только спустя эти 10 секунд, это всё о привязке времени к пространству. но мысль сообщения вот в чём, если настолько малые частицы как кванты неподвластны времени, что если один-единственный квант может находиться в один и тот же миг во всех частях вселенной при этом в разных состояниях и формах, порождая этим самым собственно все вещества и материю галактики биологию химию нас с вами и вообще всё, всё это одна-единственная частица существующая вне привязки ко времени в силу своей бесконечно малой величины. и что если биологическая жизнь и наше сознание, то что мы с вами изучаем вселенную, реальность окружающую нас, ну и вообще живём и наблюдаем/ощущаем наш мир, может на самом деле это просто попытки этой частицы познать саму себя и когда мы думаем что познаём мир на самом деле это она через нас познаёт свою природу. ну вот, такая наркоманская теория:D
317583317587
#233 #317576
На Ио есть тектоника плит? Почему у него нет мощной атмосферы, если он постоянно выбрасывает тяжелые газы вместе с извержениями?
317580317581317582
180 Кб, 1280x960
#234 #317577
Где-то читал, что гравитационные волны могут быть использованы для передвижения и что некоторые черные дыры могли быть выброшены из ядер галактик за счет гравитационных волн? Как это работает?
317593
#235 #317578
Можно ли нагреть вакуум?
317579317589
#236 #317579
>>317578
смотря насколько вакуум
#237 #317580
>>317576
вторая космическая маленькая, выбросы шустрые
#238 #317581
>>317576

> Почему у него нет мощной атмосферы, если он постоянно выбрасывает тяжелые газы вместе с извержениями?


алсо выдувает радиационным поясом
#239 #317582
>>317576
Нет данных, есть ли на Ио плиты вообще. Судя по внешним признакам - нет.

>Почему у него нет мощной атмосферы


Потому что притяжение смехотворное, все что Ио выблевал через вулканы уносится нахуй заряженными частицами магнитосферы Юпитера.
#240 #317583
>>317575
Хороший барыга у тебя, годный стафф толкает.
#241 #317584
>>317574
Могу порекомендовать Universe Sandbox. Там и сам можешь попробовать
#242 #317587
>>317575

>Привет, такой вопрос: может ли оказаться что всё что мы видим, вся реальность/материя и т.д. всё это одна-единственная частица?


Дальше не читал и можно было не читать, собственно таких гипотез немало - что наблюдаемая Вселенная может быть содержимым черной дыры, одной частицей, белой дырой и т.п.

Правильный ответ: может. А может и не может. Ибо всё это не проверяемо и не доказуемо. Хотя некоторые сходства между свойствами Вселенной, частиц и черных дыр действительно существуют. Но могут оказаться и сходством между акустическими и электромагнитными волнами, например, т.е. описываться схожим матаппаратом, не более.
#243 #317588
>>317569
>>317571
Бамп вопросу, огурцы
#244 #317589
>>317578
Можно наполнить его излучением. Это не будет температура в бытовом смысле, впрочем, т.к. она описывает состояние вещества.
#245 #317590
>>317573

>Можно заглушить GPS из космоса?


Можно глушить приёмники, направляя сигнал на Землю и ухудшая сигнал/шум на каналах L1-L5, как это делают обычные глушилки. Можно спуфить гражданский сигнал (SPS) на территории покрытия транспондера. Военный (PPS) не выйдет, ибо A/S и шифрование. Тут все зависит от орбиты и мощности ЯЭУ. Чтобы глушить постоянно на некоторой территории - нужна стационарная орбита, а это 36000км и заебёшься фокусировать, тем более не на экваторе. На низкой орбите пролёты слишком редки.

>Тогда спутники не получая сигналов сами начнут падать?


Падать не начнут, лол. Если речь о том, что наводиться на сами спутники, оставив те без линка с Земли - то GPS не зависят от 100% присутствия КРЛ, слабо представляю как им это помешает. Да и Block III имеют узконаправленные антенны, им на это решительно похуй.

Кроме того, есть еще и другие системы навигации.

Насчет того что на картинке: это видимо просто первое, что пришло в голову дуболомам "куда бы нам захуярить эту хуйню, что сейчас делают". Такой аппарат неподъёмен для отрасли в её текущем виде, это же на порядок круче чем топовые телекомовские ГСО-ёбы, даже если реактор опустить, а они в такое не могут. Реализовывать это всё равно будет кто-то с опытом построения живучих ИСЗ, а кто это может быть? ИСС?

Гораздо более реалистичен вариант системы а-ля Легенда для такой установки, к примеру. Имея источник бортпитания хорошей мощности, можно и повыше висеть и САС побольше обеспечить.
317749
5104 Кб, 4000x2666
#246 #317593
>>317577
Как сёрфинг.
#247 #317601
Можно плевками или как-нибудь иначе создать себе хоть какую-то тягу в невесомости? Читал тут фрагмент из книги и не верится. Вот фрагмент

>он сказал однажды, что идеальное убийство с легкостью можно совершить на орбите и сомнительно, чтобы какой бы то ни было суд мог осудить убийцу: достаточно уговорить жертву раздеться догола перед душем и чуть толкнуть ее так, чтобы она зависла посреди помещения и извивалась там, пока не умрет с голоду, а перед судом можно признаться, что пошел за полотенцем и забыл о нем.

317602317603317606
#248 #317602
>>317601
Хуйня, можно и дуть и махать руками, просто будет долго и неудобно, вот и всё. Алсо, в любых помещениях для невесомости по-любому будут предусмотрены поручни или еще какие цеплялки, чтобы не барахтаться как дурак.
#249 #317603
>>317601
Выдохнуть сильно в одном направлении.
317605
#250 #317605
>>317603
А если поссать, ощутимая тяга будет?
317607
#251 #317606
>>317601
хуйня лютая. чуть толкнув, нельзя повесить тело в невесомости. оно будет продолжать двигаться в направлении, куда его толкнули. даже если его не толкнуть а поместить, все равно вероятность не зарядить никакого импульса - ничтожна. и тело рано или поздно прибьет к стене.

даже если телу надоест дрейфовать, оно всегда может, действительно, дуть, задрав голову вверх вдоль оси тела (иначе оно начнет сильнее вращаться). медленный вдох и усиленный сигаретный выдох, сложив губы жопкой соплом - реактивная тяга, лол.

если предпочтительнее движение головой вперед, то плавательные движения ладонями плавно вверх к голове и резко от лица и вдоль тела вниз - так легче всего вести руки по продольной траектории и наиболее эффективно расходовать силы, не растрачивая их на поперечные компоненты гребка.

через пару минут такой гимнастики и пранаямы вполне реально разогнаться и успеть отдохнуть, плавно плывя к твердой поверхности, а дальше толкаться от стен и плыть напинать обидчика.

алсо, немного на эту тему было в книжной версии Экспансии.
#252 #317607
>>317605
Безусловно, это же выброс рабочего тела. Реактивное движение!
#253 #317613
Объясните довну, как космонавты и марсоходы снимают себя, при этом камера не закреплена на парсоходах и её нет в руках, но при этом следует за ними?
317615317637
#254 #317615
>>317613
Например?
Если ты "Марсианина" имеешь ввиду то этого не было на самом деле. Художественная выдумка.
317650
#255 #317623
У меня такой вопрос: как срать в условиях невесомости? Я помню, искал ответ в интернете, люди предлагали разные варианты, даже есть пищу богатую железом и срать в магнитный унитаз.
317624317625
#256 #317624
>>317623
вакуумный опорожнитель кишечника
317626
#257 #317625
>>317623
как и всё прогрессивное человечество на МКС - срать в пылесос
#258 #317626
>>317624
Это шланг нужно засунуть в жопу?
317630
#259 #317630
>>317626
Тебе лишь бы засунуть что-то в жопу. Прикинь шланг там к кишке присосется.

Нет, конический аппликатор, который прислоняется к анусу и создает отрицательное давление.
317631
#260 #317631
>>317630
А если понос? Всё же может разлететься, скорости всасывания может не хватить
317635
#261 #317635
>>317631
Но как-то же космонавты писяют.
#262 #317636
Останавливается ли время внутри чёрной дыры?
317651
3593 Кб, 3840x1296
#263 #317637
>>317613
Космонавтов фотографируют другие космонавты.

У ровера камера находится на манипуляторе, который имеет много степеней свободы, камерой делается серия снимков, которые потом склеиваются в один.
https://www.instagram.com/p/6k4pUMFbys/
#264 #317638
>>316903
Двачую анон. Тоже поражаюсь возможной бесконечности вселенной.
1. Гипотетически поверхность сферы для двумерного наблюдателя на ней бесконечна. Но это не так для трехмерного наблюдателя. Врямя тоже измерение? Ок, и что? Просто если брать х+1 измерение, то не окажется ли бесконечность в х конечной в х+1? Кол-во измерений бесконечно или упираемся в 10ое-11ое?
2. Придумали какие-то брамы хуямы. Мультивселенные сюда же. Их тоже бесконечное количество? ГДЕ они находятся? То ГДЕ они находятся тоже бесконечно? То ГДЕ они находятся одно или тоже мульти? Если мульти, возникают те же вопросы. И т.д.
317641
100 Кб, 610x395
#265 #317640
Как разобраться в положении астрономических объектов в нашей галактике? Ну вот есть у меня "карта" Млечного Пути, на которой нихуя кроме рукавов не отмечено, есть карты звёздного неба, как мне теперь перенести второе на первое?
Ну или может есть пиздатые карты галактики у кого, где помимо рукавов и Солнца какие-нибудь "достопримечательности" обозначены.
317644317648
#266 #317641
>>317638

>2


К сожалению, параллельные миры это сорт оф математическая абстракция.
75 Кб, 640x589
#267 #317642
Если я выйду из МКС в открытый космос с АК-74, и начну стрелять в направлении орбиты, хватит ли у вылетевшей пули скорости стать спутником Земли и вращаться по орбите?
А если я, боясь что меня отбросит назад, поступлю как в первом случае, но держась за МКС или стреляя в гипотетическую щель форточки иллюминатора, повторю действие из первого пункта, да не одним выстрелом, а чредой, сильно ли это изменит вращение, наклон и саму орбиту МКС?
#268 #317643
>>317642

>Если я выйду из МКС в открытый космос с АК-74, и начну стрелять в направлении орбиты, хватит ли у вылетевшей пули скорости стать спутником Земли и вращаться по орбите?


Если ты вышел из МКС, то можно не стрелять, вынуть пулю и отпустить, у нее уже достаточно скорости, что бы стать спутником Земли.
133 Кб, 1200x1196
#269 #317644
>>317640
Ну это дело не хитрое, держи пикрелейтед.
#270 #317645
>>317642

>А если я, боясь что меня отбросит назад, поступлю как в первом случае, но держась за МКС или стреляя в гипотетическую щель форточки иллюминатора, повторю действие из первого пункта, да не одним выстрелом, а чредой, сильно ли это изменит вращение, наклон и саму орбиту МКС?


Посчитай. Масса мкс 400 тонн скорость пули 870 метров в секунду(в среднем) масса пули около 4 грамм(если ничего не путаю). Гули, считай, удачи
317647317649
#271 #317647
>>317645

>870 метров в секунду


Вангую, в вакууме она будет выше.
#272 #317648
>>317640

>Как разобраться в положении астрономических объектов


Скачать Стеллариум.
#273 #317649
>>317645
скорость пули немного больше будет)
-1 атм внешнего давления.
360 Кб, 1000x688
#274 #317650
>>317615
Про космонавта не найду, он там по ТВ был, обследовал купол на МКС, камера типа как селфипалка за ним в этот закуток следовала, но понятно что он там один был, но палки не было.
317652
#275 #317651
>>317636
Как говорит нам дедушка Альберт, время - это свойство самого вещества, а не внешнее поле. А так как чёрная дыра по сути одна большая элементарная частица, то очевидно, что вопрос некорректен.
317654
#276 #317652
>>317650

>но понятно что он там один был


Ты примерно так почувствовал? По одному на ВКД не ходят.
317653
#277 #317653
>>317652
Он показал это окно и перевернув каиеру к себе еле туда влез, тоесть камера держалась как будто он её держит, а не другой человек или она дрейфовала в невесомости, но при этом монопода небыло в руках. Ну блядь, что я объясняют, дебилов с селфипалками не видели что ли?
#278 #317654
>>317651
Чд это объект в пространстве, еблон. Или у тебя и кирпич частица?
317664
#279 #317658
>>317642

>Если я выйду из МКС в открытый космос с АК-74, и начну стрелять в направлении орбиты


Застрелишься, что ли?
#280 #317659
>>317642

>сильно ли это изменит вращение, наклон и саму орбиту МКС


Нет. Практически никак.
11 Кб, 225x225
#281 #317662
Если Плутон не планета, то откуда у него аж целых 5 спутников? Чет у метеоритов или какой нибудь луны нет спутников.
317663317706317710
#282 #317663
>>317662

>то откуда у него аж целых 5 спутников?


Ну, предположим, появились в результате столкновений.

>Чет у метеоритов или какой нибудь луны нет спутников.


Ну и хуйли? Плутон не астероид и не луна.

>у метеоритов нет спутников.


Метеорит - это ебаный камень, или кусок железа лежащий в грунте, у него не может быть спутников по определению.
317701
#283 #317664
>>317654
Сам еблон. Элементарная частица не является объектом в пространстве?
317666317667
#284 #317666
>>317664
Проблема в том, что ЧД - это область пространства-времени, имеющая границы по горизонту событий, чего о частице ну никак нельзя сказать.
#285 #317667
>>317664
Ебало на ноль, пидарина. Иди хоть педивикию читни, частица у него размером в несколько световых лет.
317668
#286 #317668
>>317667
Туда же, пидор.
#287 #317671
С точки зрения Теории Струн ЧД - это тоже частица
317672
#288 #317672
>>317671
Нет, волна.
317673
#289 #317673
>>317672
Какая ещё, в жопу, волна?
317674
#290 #317674
>>317673
Электромагнитная.
#291 #317679
Спейсаны, поясните за временной барьер, вот тут прочитал http://secrets-world.com/space/1012-vremennoy-barer.html но нихуя не понял, какие-то вероятности, 3,7 миллиарда лет, охуеть вообще. Поясните короче тупому в чем его суть и где подробнее почитать.
317680
#292 #317680
>>317679

>АНОМАЛИИ


>НЕПОЗНАННОЕ


>ПРОРОЧЕСТВА


>СВЕРХЪЕСТЕСТВЕННОЕ


>ТАЙНЫ ИСТОРИИ


Ты явно не туда зашёл. Причём дважды.
317683
#293 #317683
>>317680
Блядь, нормально ответить можешь? Я вроде не в /b/ пришел
317684317698
#294 #317684
>>317683
-> /zog/
317685
sage #295 #317685
>>317684
Ебать, да идите вы нахуй, аутисты, пришел блядь с небольшим вопросом, можно ж просто ответить парой предложений, нееет, не хочу, хочу выебнуться на анонимной доске!
317686
#296 #317686
>>317685

> мам, я тралю спейсач

317688
sage #297 #317688
>>317686

>мам я пишу гринтекстом и тонко траллю как настоящий ононимус ну маааам

#298 #317698
>>317683
У нас тут своя атмосфера и наукообразная чушь это не к нам. Конкретно по ссылке - поток сознания уровня электрической лампочки от 120 кирпичей, его нет смысла комментировать.
#299 #317701
>>317663

> Метеорит - это ебаный камень, или кусок железа лежащий в грунте, у него не может быть спутников по определению.


Скажи это Розетте, смело вращавшейся на орбите кометы Чурюмова-Герасименко.
317702317706
#300 #317702
>>317701
Это уже даже не смешно.
#301 #317703
>>317139
Станция new horizons пиздячила до плутона 10 лет. И это без людей, где можно не учитывать туеву хучу нюансов человеческого организма и пиздячить лююое ускорение с минимальным грузом. 10 лет туда, 10 лет назад. Итого сейчас просто космическому аппарату понадобиться 20 лет на этот твой полет

>обыденностью как и полёт в другую страну.

90 Кб, 664x456
#302 #317706
>>317701

> метеорит


Пикрилейтед.

Алсо, у Розетты была квазиспутниковая орбита, корректируемая движками. А иногда и прямо формируемая ими. Там не было традиционной орбиты вокруг кометы.

>>317662
Столкновения, баллистический захват. Хаумея имеет как минимум два спутника, например, и вообще не исключено что это типично для транснептуновых объектов. Существуют даже тройные астероиды, если что. Даже та же 67P слиплась из двух говёх.
#303 #317710
>>317662
Может себе позволить. Обладает достаточной массой для захвата, в придачу "повезло" с орбитой для этих целей. Никакие надоедливые газовые гиганты своим вмешательством не напрягают. У Луны нет спутников, потому что мамка Земля не позволяет в основном. Собственно технически спутников наверняка много - куча каменюк, возможно более метра диаметром, делающих много оборотов, просто их орбиты не стабильны и их ждет бесславный конец. Но если бы Луна вращалась в гордом одиночестве на земной орбите, то вокруг нее вполне могли бы крутиться довольно крупные объекты.
624 Кб, 2797x1886
#304 #317741
По вашему мнению, будут ли когда-нибудь возможно межгалактические полёты? И будут ли в качестве топлива использоваться простые природные ресурсы, либо же надо будет изобретать нечто более сложное?
317745
185 Кб, 160x160
#305 #317744
>>317400

>вселенная не расширяется


>звёзды горят стабильно с одной температурой вечно


>что произойдёт с жизнью


И ТУТ ВСЕ НАХУЙ БЛЯДЬ СГОРЕЛИ В СТРАШНЫХ МУЧЕНИЯХ ОБУГЛИЛИСЬ ВОЛДЫРЫ КОЖА ЛОПАЕТСЯ ГНОЙ ТЕЧЕТ ГЛАЗА ЗАКИПАЮТ СУКА БЛЯДЬ ХУЛИ ТЫ ТАКУЮ ХЕРОВУЮ ВСЕЛЕННУЮ ПРИДУМАЛ ПИДР
https://ru.wikipedia.org/wiki/Фотометрический_парадокс
135 Кб, 1024x576
#306 #317745
>>317741
Они уже возможны. Все пришельцы уже улетели нахуй из нашей галактики в великую IC 1011.
317750
#307 #317746
Может так получиться, что мы никогда не узнаем причину Большого взрыва? В смысла, что информация навсегда потеряна, как в черной дыре и никак не восстановить, что же произошло во время и до Большого взрыва?
#308 #317747
Можно ли из космоса как-то ловить теряемый планетой газ в промышленных масштабах? Типа сделать гигантские спутники с щупальцами длинной в тысячи километров, которые улавливали срываемый газ в магнитных линиях и консервировали его?
#309 #317749
>>317590

>Можно глушить приёмники, направляя сигнал на Землю и ухудшая сигнал/шум на каналах L1-L5, как это делают обычные глушилки.


А можно использовать как-нибудь свойства ионосферы и увеличивать покрытие помех за счет отражения?

>реалистичен вариант системы а-ля Легенда для такой установки, к примеру.


а можно поподробней?
#310 #317750
>>317745
Нифига там туса. А нас не позвали. Пидарасы.
317752
#311 #317752
>>317750
[Эдмонд Гамильтон mod on]
Они эту мегагалактику создали из мелких галактик в те времена, когда еще Солнце не родилось. Сейчас уже даже следов от столкновений не видно, она приобрела спокойную эллиптическую форму.
Зато мы с Андромедой объединимся и быть может наше столкновение это тоже часть чьего-то плана по созданию мегагалактики.
[Эдмонд Гамильтон mod off]
#312 #317757
А планеты вообще тормозят из-за солнечного ветра и излучения? Ну, как спутники тормозят в атмосфере о частицы. например Меркурий вообще близко к Солнцу, может даже в протуберанцах купается.
234 Кб, 1138x1145
#313 #317776
Посоны, почему на Марсе нет магнитосферы но нет и радиации сильной?
317777317788
#314 #317777
>>317776

> нет и радиации сильной?


в сотню-другую раз больше чем на землюшке
хотя атмосфера немного гасит, это да
317778318098
#315 #317778
>>317777
О какой колонизации может тогда речь идти?
317779
#316 #317779
>>317778
так это без скафандра даже, в будущих зданиях и под землёй вообще норм
алсо схожий фон на МКС - брат жив, зависимость есть
317785
89 Кб, 481x594
#317 #317785
>>317779

>брат жив

317787
#318 #317787
>>317785
ты понял о чем речь, да)
#319 #317788
>>317776
Магнитосферы нет, потому что железий в ядре не крутится.
Есть магнитосфера = ионы с электронами крутятся в ней, и радиация будет на орбите.
Нет магнитосферы = ионы с электронами ебашат тебя по голове, и радиация будет на поверхности.
317791
#320 #317791
>>317788
А можно чем-нибудь ещё покрутить шоб магнитосферу завести?
317792
#321 #317792
>>317791
Тут мамкины терра-формисты предлагали обруч как на бочку и резко потянуть.
317793317827
#322 #317793
>>317792
А если провод намотать и ток пустить?
317794
#323 #317794
>>317793
Нужно придать вращение каким-то образом.
317795
#324 #317795
>>317794
Ну в смысле катушку вокруг Марса накрутить. Должно же магнитное поле появиться?
317796
#325 #317796
>>317795
ну, если намотать охлаждаемый сверхпроводник и подать ток - должно выйти
мотать только дохуя прийдётся
317798
#326 #317798
>>317796

> мотать только дохуя прийдётся


Дохуя - это сколько? Обязательно использовать сверхпроводники?
317800
#327 #317800
>>317798
это уже не совсем тупые вопросы
#328 #317812
Пейсач, а как отводится тепло в космосе, там же нет передающей среды. Условно есть но частиц совсем мало на единицу объема.
317813
131 Кб, 990x1066
131 Кб, 650x450
322 Кб, 640x368
#329 #317813
>>317812
Тепловое (инфракрасное) излучение.
317821
#330 #317821
>>317813
Блин всегда мечтал себе купить тепловизор. Еще со времен просмотра робокопа в 90-х.
317998
#331 #317827
>>317792
Вообще для терраформирования более пригодна Венера. У неё есть магнитное поле. Воды нет. Атмосфера состоит, в основном, из углекислого газа, но на высоте 80 км находится плотный слой облачности из паров серной кислоты, надо сказать, у поверхности серной кислоты почти нет. Есть идея кинуть на венеру несколько больших астеройдов состоящих из воды и аммиака. Вода и углекислый газ - среда для развития бактерий даже при 450`C. Эти бактерии переработают углекислый газ в кислород и понизят температуру. Правда, непонятно, что произойдёт с облаками серной кислоты. В принципе она может прореагировать с аммиаком, и получится даже удобрение.
317828317829317831
#332 #317828
>>317827

>>Вода и углекислый газ - среда для развития бактерий даже при 450`C



Таки нет. Максимальная температура, при которой известные на сегодня экстремофилы размножаются - 122 градуса. Выше 150 градусов жизнь, использующая ДНК невозможна, т.к. это температура плавления ДНК.

Биолог на мимопролётной траектории
16 Кб, 320x256
#333 #317829
>>317827

>У неё есть магнитное поле.


Ты не охуел ли путать магнитное поле планеты с индуцированной магнитосферой? Из-за этого говна у нее в атмосфере и воды нет нихуя.
#334 #317831
>>317827
Атмосферу куда девать будешь?
317834
10110 Кб, Webm
#335 #317832
Меня пугает эта вебмка.
317833
#336 #317833
>>317832
С 1.12 у меня всегда мурашки бегут по спине, алсо это одна из тех вебмок которые если отрыл то всегда смотрю до конца.
#337 #317834
>>317831
Насосом на Марс отсосу.
146 Кб, 1200x1200
#338 #317835
Привет анон. Собрался смотреть "космос" тайсона. Стоит ли сперва заценить кинцо сагана? Или получится одно и то же, только с разным графоном?
317898
#339 #317898
>>317835
Привет. "Космос" Сагана сейчас имеет в основном историческое и культурное значение. Сериалу хуева тьма лет и многие данные откровенно протухли, какие-то теории пересмотрены, что-то было пересмотрено и уточнено. Сам Саган охуенен, да Так что можешь ознакомится, конечно, но смотреть нужно "Космос" с Тайсоном. Там и графоний завезли.
#340 #317989
Вопрос не в тему треда. Но все же. У меня одного дико печет что под видосах о Космосе и Вселенной, обязательно найдется кто напишет "как велик наш Создатель" и т.п.
Пиздец как печет.
#342 #317991
>>317989
Типнул федору.
#343 #317998
>>317821
300к всего, копи, покупай.
#344 #318006
>>317989
А у меня печет от атеистов-борцунов, которые начинают срач под такими комментариями.
мимо атеист
318021
#345 #318009
У Леонова при возвращении на корабль возникла проблема из-за раздувшегося скафандра. Он не сразу смог влезть шлюз, поэтому пришлось сбросить давление. Почему эта проблема возникла только при возвращении, а не в начале выхода, когда космонавт пробирался через разгерметезированный шлюз к открытому космосу?
318019318022
#346 #318019
>>318009
Это как лампочку в рот вставлять.
318020
#347 #318020
>>318019
Но почему же при этом интересном трюке обычная лампочка легко входит в рот, а потом чувствует там себя как дома и совсем не хочет выходить? Оказывается все просто. Так как лампочка имеет обтекаемую грушевидную форму, то она легко «проскальзывает» в рот, а вот обратный процесс с подобным скольжением не связан. Тем более, что при удержании во рту лампочки мышцы напрягаются и челюсти сжимаются еще больше, и лампочка находится в сильном обхвате, который не так то просто разжать.
Вообще не то. Мышцы шлюза напрячься не могут ввиду их несуществования. И навстречу космосу Леонов не выскальзывал.
318022318027
sage #348 #318021
>>318006
Ты пидор.
>>317989
И ты.
318046
#349 #318022
>>318009
А он и при выходе внатяг сидел. Просто стенки шлюза не давали скафандру расшириться.
>>318020
Ты щас какие-то анальные игрища описал.
318024318027318028
#350 #318024
>>318022

>стенки шлюза не давали скафандру расшириться


Был зазор между стенками и скафандром. внутренний диаметр шлюза – 1 метр, ширина скафандра в плечах – 68 см
http://www.aif.ru/society/science/progulka_v_nevesomosti_kak_aleksey_leonov_vpervye_vyshel_v_otkrytyy_kosmos
318028
#351 #318027
>>318020
>>318022
Леонов с ощутимым сопротивлением вошел в напряженный сфинктер шлюза Восхода-2 и начал там переворачиваться.
26 Кб, 659x366
#352 #318028
>>318022
>>318024
Из фильма про выход. Он свободно проходил в вакууме, почему не раздулся?
318030
#353 #318030
>>318028
Не успел надышать в скафандр достаточно воздуха, что бы тот раздулся.
318045
#354 #318045
>>318030
Воздух же из баллонов подается под давлением 0,4 атмосферы,а не из легких. К тому же отработавший воздух постянно удаляется сквозь вентяляционный клапан.
318051
#355 #318046
>>318021
Нет ты
#356 #318051
>>318045
В целом возникшие трудности можно объяснить тем, что методика входа в шлюз была недостаточна, отработана в наземных условиях. Кроме того, как неоднократно рассказывал после полета сам А. Леонов, он пытался войти в шлюз вперед головой, а не ногами как отрабатывалось на Земле, в результате чего ему пришлось уже внутри шлюза переворачиваться для входа в СА. Эти затруднения могут быть объяснены необычными условиями открытого космоса и невесомости, которых не было при наземных испытаниях.
318054
#357 #318054
>>318051
Здесь написано про трудности возвращения в шлюз, но не про выход из него, что меня собственно интересует
318055
#358 #318055
>>318054

> Здесь написано про трудности возвращения в шлюз, но не про выход из него, что меня собственно интересует


> У Леонова при возвращении на корабль возникла проблема из-за раздувшегося скафандра. Он не сразу смог влезть шлюз, поэтому пришлось сбросить давление. Почему эта проблема возникла только при возвращении


так че тебе там надо было?
318102
#359 #318098
>>317777
Илон Маск летал радиации не видал.
#360 #318102
>>318055
мой вопрос(твое цитирование) начинался с вводной о проблеме Леонова влезть обратно в шлюз.
Потом я задал непосредственно вопрос - как же вначале он вылез из шлюза без проблем?
Схема выхода.
1.Открывается внутренний люк между шлюзом и герметизированным пространством корабля.
2. Леонов заползает в шлюз.
3. Беляев, закрывает внутренний люк, Леонов подключается к системам жизнеобеспечения скафандра.
4. Беляев разгерметизирует шлюз. В шлюзе образуется вакуум
5. Беляев открывает внешний люк между шлюзом и космосом.
6. Леонов без проблем выходит из шлюза в открытый космос.
7. Скафандр в вакууме космоса раздувается.
8. Спустя 12 минут Леонов с огромным трудном вползает обратно в шлюз.
Вопрос почему скафандр раздулся на 7 пункте,
а не на 4?
318103
#361 #318103
>>318102
Потому что блядь скафандр имеет один размер что в вакууме, что на Pемле, про раздувшийся скафандр - это маняистория. Леонов лично блядь сам сказал, что возвращаясь полез в шлюз башкой вперед, а надо было ногами вперед заходить. Когда начал в этой хуйне переворачиваться, то закономерно застрял, ему пришлось стравить воздух в скафандре, что бы скафандр мог лучше сгибаться, потерял в объеме, и проще было перевернуться. Вот отсюда история про якобы раздувшийся сверхмеры скафандр.
318148
#362 #318105
Сап космач, вопрос на ноль - какой мощности должен быть эми со стороны Солнца, чтобы сжечь всю электронику на Земле? И даже под землей? И какой вред здоровью будет нанесен местной живности (хуманам в том числе)?
318111
#363 #318106
Делаем логично как в математике 1 - 1 = 0.
Материя - антиматерия = ничего.
Почему не сделать прибор, который из пустоты бы не делал материю и антиматерию?
Из материи можно было бы строить дома, а антиматерию использовать как топливо для звездных перелётов.
318107318637
#364 #318107
>>318106

>1 - 1 = 0


Пиздёж. Образуется электромагнитное излучение.

>Почему не сделать прибор, который из пустоты бы не делал материю и антиматерию?


Это не так просто, ка тебе кажется.
В принципе, ещё сотню лет назад было показано, что ОЧЕНЬ высокоэнергетическими гамма-лучами прири бомбардировке тонкой фольги порождаются пары электрон-позитрон. Но на это требуется дохуя энергии. Где ты её возьмёшь?
318108
#365 #318108
>>318107
Холодный синтез же или ЭмДрайв.
318109318114
#366 #318109
>>318108

>ЭмДрайв


Значение знаешь?
318110
#367 #318110
>>318109
Как правильно? Я недавно читал что НАСА подтвердило что ведро работает.
318112
11534 Кб, Webm
#368 #318111
>>318105
Какой нахуй ЭМИ? Солнечная вспышка может максимум попортить спутники на орбитах, Солнце не может создать вспышку такой силы внезапно, которая пожжет электронику на Земле.

Проблемы доставляют геомагнитные бури, причинами которых являются выбросы корональной массы. Когда Солнце выбрасывает корональное вещество, которое состоит из протонов и электронов, в основном, плюс ядра и осколки ядер. Вся эта ебала, если устремляется в сторону Земли может реально наделать бед.
Например событие Кэррингтона в результате которой погорели телеграфы по всему миру, это в 1859 году, когда люди электричество почти не использовали, случись сейчас событие с Dst в 700-900 нТл - настал бы пиздец любой энергосистеме. В 1989 геомагнитная буря Dst 500-600 нТл вызвала помехи в радиосвязи, и пизданула энергосистему систему в США и Канаде. В 2012 геомагнитная буря опять таки уебала по энергосистеме США, что стоило им от 0.5 до 2.5 триллионов баксов.
Но выброса корональной массы, что бы пожечь нахуй всю электронику вообще всю, даже под Землей - не приходится, так же как каких-то серьезных последствий для биосферы. Солнце, как желтый карлик - относительно стабильный класс звезд, это не в их природе жечь планеты выбросами и вспышками. Так что при самом хуевом раскладе максимум придется в темноте несколько дней посидеть и ойфон с пекой новый купить.
#369 #318112
>>318110
Это, блять, ДВИГЛО. Оно не выделяет энергию, а только жрётспит и срёт.
318113
#370 #318113
>>318112
Да? А читал где-то что он из нихуя делает немного энергии, видимо читал через строку.
#371 #318114
>>318108

>Холодный синтез


Пиздуй нахуй отсюда со своей маняхуетой.
#372 #318148
>>318103

> про раздувшийся скафандр - это маняистория


Об этом рассказывал лично Леонов. Получается, он соврал.

>Леонов лично блядь сам сказал, что возвращаясь полез в шлюз башкой вперед, а надо было ногами вперед заходить


Все стало еще страньше. На съемках Леонова четко видно, что возвращается он вперед ногами.
37:42-37:56 здесь это видно https://www.youtube.com/watch?v=tXu_dlVVGNo
#373 #318151
Спейсаны, а есть какие книги/видео про устройство всяких спутников и аппаратов в деталях (от агрегатов до программного обеспечения)?
318152318156
#374 #318152
>>318151
Это военная тайна, еблоид.
318153318156
#375 #318153
>>318152
Даже телевизионных спутников?
318154
#376 #318154
>>318153
Коммерческая тайна.
318155318156
#377 #318155
>>318154
Сраный капитализм
#378 #318156
>>318151
Что конкретно тебя интересует? С английским дружишь? Простого и обзорного ничего не знаю.

>>318152
>>318154
Это тред тупых вопросов, а не тупых ответов, лол.
318157
#379 #318157
>>318156
Да в принципе вообще все, просто что-то вспомнил как читал ламповую советскую книжку где было описание Луны-13, как устроенное что делает и т. п. С английским дружу
318158
#380 #318158
>>318157
Тогда читни например

>Spacecraft Systems Engineering, 4th Ed


>Planetary Landers and Entry Probes


Обе максимально компактно описывают разработку подсистем и миссий, в первой можно брать любую главу и читать, вторая просто пиздатая.
318160
#381 #318160
>>318158
То что надо, спасибо
#382 #318213
Если на Марсе найдут скелеты вымерших существ, что это изменит в программе НАСА и прочих космических агентств?
318218
sage #383 #318218
>>318213
А что может принципиально поменяться? Ну поймут они что Земля не уникальна, запустят несколько аналогичных исследовательских миссий к другим телам СС типа Европы и Титана. Может пару астероидов побурят. Правительство под давлением возбужденной общественности будет давать больше бабла на исследования космоса, что раскроет большие возможности для распилов. Через несколько лет весь этот бум спадет, быдло поймет что никакого Масс Эффекта с еблей синекожих инопланетянок не предвидится и вернется обратно запускать птиц по свиньям да выплачивать ипотеку. Возможность существования жизни на других планетах будет восприниматься как такая же норма как то, что Земля вращается вокруг Солнца. Финансирования космических программ вернутся в норму, может чуть выше чем до открытия скелетов, но мы все так же будем жечь сотни тонн токсичного горючего что бы запустить человечков в алюминиевой банке на другую, к тому времени изрядно обветшалую и завершающую свой жизненный цикл банку на орбите.
#384 #318272
>>316443 (OP)
Прочитал тред, где анон спрашивает про то, как построить ракету. Он уже полумёртвый тред, с аноном, надеюсь, всё в порядке, поэтому спрошу тут.
Как я понял, то ГИРДовцу в России тяжело — законодательно никакие свои изыскания не оформишь. На что-то уровня фейерверка, конечно, глаза закроют. Но начиная с определённой высоты у органов могут возникнуть вопросы. Даже вполне обоснованные. Ибо есть самолёты, а ещё ракета может упасть на военную часть или ещё куда. И будет студент объяснять потом, что ему просто интересно.
В связи с чем вопрос: какие страны представляют больше возможностей для таких любителей? И как это оформлено? Ведь для пуска нужно выбрать место, где он со своей сарайной ёбой никому не навредит.
318273
sage #385 #318273
>>318272
У нас есть http://www.frms.ru/ Федерация ракетомодельного спорта России. Никаких законов, которые ограничивают ракетостроение, не существует. В законах прописаны только БОЕВЫЕ ракеты, если у ракеты нет боевой части, то она под ограничения не попадает. Что бы запустить ракету нужно во-первых согласие владельца земельного участка, с которого будет пуск в письменном виде, плюс за пять дней нужно подать заявку в Росавиацию, к заявке нужно приложить это самое разрешение от владельца земли. В Росавиации, кстати, в курсе о любителях ракетного моделизма, там ты этой заявкой никого не удивишь. Плюс потребуется страховка гражданской ответственности, на случай, если ракета кому-то пизданет на голову, или повредит собственность. В страховых конторах похуй что страховать, но счет могут выкатить нормальный.
Поэтому лучше всего договариваться с военными на полигонах, в Московской области такие прецеденты уже были. Этим страховка не нужна, они проинспектируют место пуска, пусковые и ракеты, и будут присутствовать на запусках, в качестве наблюдателей-допускающих.

Проще всего с этим делом в США, там проводятся крупные фестивали ракетчиков-любителей. Двигатели и топливо продают в магазинах на любой вкус и кошелек. У них есть специальные места, где проводятся запуски, так что там все происходит в уведомительном порядке. С другой стороны, там надо очень аккуратно относиться к таким вещам, так как шаг вправо-шаг влево от регламентов и законов все это регулирующих и можно влететь на бабки, или на сгуху. Так что нужно знать, как это регламентируется и как это регулируется местным законодательством. Но поскольку там этот спорт очень некисло развит, можно сделать вывод, что все это несложно.
В ютьюбе по запросу Amateur rocketry можно посмотреть, как это все выглядит.
318301
#386 #318275

>Масса мкс 400 тонн


Вот. А если например на орбите собрать кучу термоядерных бомб, по типу 100Мт совейской кузькиной матери, запустить всю эту хуйню к луне и там невозбранно ёбнуть около поверхности? Что будет?
318280318282
#387 #318280
>>318275
Будет термоядерный взрыв на луне. Wait, oh shi... А ведь из луны можно сделать ядерный полигон.
318281318282
#388 #318281
>>318280

> А ведь из луны можно сделать ядерный полигон.


землюшка дешевле
#389 #318282
>>318275

>Что будет?


Нарушение договора о космосе от 1967 года.
>>318280

>А ведь из луны можно сделать ядерный полигон.


Нельзя по той же причине.
#390 #318301
>>318273
Спасибо, анон.
#391 #318302
>>316443 (OP)
на втором пике Кодзима?
318306
#392 #318304
Че там с Луной? Уже 2017, а исследовательскими автономными базами там так и не обмазались (позор). Чего боятся то? Это же изи, раза в два дешевле любой олимпиады и поднимет имидж страны
318305318307
#393 #318305
>>318304
В ближайших планах китайская АМС с посадкой на обратной стороне (вспомогательный аппарат уже пускали). Ещё несколько АМС от лавки. Может быть несколько частных, особенно если Astrobotic кому-нибудь таки сбагрит свою маняплатформу. Престиж-хуиж оставь за порогом, к тому же это сраная луна. Про обезьянок и базы вообще забудь сразу.
#394 #318306
>>318302
Нет, это его брат Хидео.
32 Кб, 420x673
#395 #318307
>>318304

>Это же изи

#396 #318309
Что из себя представляет Солнце? Из чего оно состоит? Оно твёрдое?
318310
#397 #318310
>>318309
Естественный термоядерный реактор ака звезду.
Из водорода и гелия, главным образом.
Состояние вещества в нём - плазма.
318311318312
#398 #318311
>>318310
Поясни за размеры "термоядерной зоны". Это же не все солнце?
318313
#399 #318312
>>318310
Может что-то пройти сквозь Солнце? У Солнца есть ядро, или оно однородное? Из чего состоит, если есть?
318315318316
#400 #318313
>>318311
Внутри, в ядре происходят термоядерные реакции. Все что выше - имеет задачу давить то, что внутри, что бы создать достаточно высокую температуру и давление, для запуска этих самых термоядерных реакций.
318334
#401 #318315
>>318312
Ядро конкретно Солнца состоит из водорода и гелия, при чем концентрация гелия растет. Так как водород в термоядерных реакциях превращается в гелий.

>Может что-то пройти сквозь Солнце?


Могут пройти нейтрино.
#402 #318316
>>318312
Сквозь Солнце - нет. (ну разве что нейтрино, да) Ибо очень плотное вещество (из-за сильной гравитации) и неебически сильное магнитное поле. В больших звездах, раздувшихся на поздней стадии эволюции, с этим попроще - внешние слои более разрежены. Утверждается, что во внешних слоях самых крупных сверхгигантов можно было бы хоть на самолете летать, если бы тот выдерживал условия.
318318318321318344
#403 #318318
>>318316

>В больших звездах, раздувшихся на поздней стадии эволюции, с этим попроще


Только не в ядрах, там уже другое, оче плотное вещество. Даже на излете жизни такой небольшой звезды как Солнце будет плотное углеродно-кислородное ядро.
#404 #318321
>>318316

>хоть на самолете летать


Автохуй. гравитация слишком большая, а подъёмная сила лимитируется скоростью (читай - сопротивлением среды)
Алсо, когда водород закончится, Солнце ебанёт?
318322318332
#405 #318322
>>318321

>Алсо, когда водород закончится, Солнце ебанёт?


Раздуется примерно до орбиты Земли, плюс-минус лапоть. Сбросит оболочки и останется белым карликом, диаметром 10000-14000 км. Составом в основном из углерода и кислорода.
318323
#406 #318323
>>318322
Жаль. Могло бы и ебануть.
318324
#407 #318324
>>318323
Солнце сильно массы недобрало для этого.
#408 #318332
>>318321

>Автохуй. гравитация слишком большая, а подъёмная сила лимитируется скоростью (читай - сопротивлением среды)


nyet, в том и дело - сверхгигант очебольшой (радиус орбиты наших внешних планет, грубо говоря), и g там вполне приемлемый, плюс внешние слои крайне разрежены, что-то вроде атмоферы Земли
318333
#409 #318333
>>318332
Солнцу не быть сверхгигантом. Поэтому

>радиус орбиты наших внешних планет


ему не светит. Максимум орбита Земли, или немного дальше.
318337
#410 #318334
>>318313
Как хитро сконструированно.
318335318338
238 Кб, Webm
#411 #318335
318339
#412 #318337
>>318333
Ну речь про них шла. Бетельгейзе всякие и т.п.
318343
#413 #318338
>>318334

> летали говна посреди нихуя


> хуяк слиплись


> сконструировано

318339
#414 #318339
>>318335
>>318338
Прост на мой дилетантский взгляд такие вещи кажутся творением кого-то. Я прост туповат немного, не бейте, лучше обоссыте.
318341
#415 #318341
>>318339
Снежинки у тебя тоже творение кого-то?
318342
#416 #318342
>>318341
а то
#417 #318343
>>318337
Но речь-то про Солнце была, зачем ньюфага путать?
318344
#418 #318344
>>318343
Ты прост в глазницы долбишься, аноним, там не в чем путаться, все ясно-понятно >>318316
79 Кб, 1000x1000
#419 #318365
Я тут двигатель придумал:
перцы - магниты, которые работают от тока
жирная точка по середине - вал
звёздочка - магнитный элемент

Суть в том, что сначала подаёшь немного энергии в систему, и магниты начинают поочерёдно включаться, тем самым раскручивая вал, дальше система изолируется от внешней энергии, и магниты ебланят сами на энергии кручения вала.

Где я обосрался?
318366
#420 #318366
>>318365
В законе сохранения энергии. И в разделе. И в выборе травы.
318367
#421 #318367
>>318366

>В законе сохранения энергии.


Так магниты не юзают много энергии, им немного тока подавай, чтобы они магнитили.

>И в разделе.


Обычно тут спрашиваю, мне отвечают, ну или гонят тряпками.

>И в выборе травы.


Не курю, хотя я сегодня плохо спал, да.
318368318374
#422 #318368
>>318367

>ну или гонят тряпками


Сейчас как раз такой случай.
318369
20 Кб, 400x386
#423 #318369
>>318368
Жаль, такой потенциал, вероятно обо мне потом будут писать в книгах, как о создателе идеи нового типа двигателей, которые повсеместно бы применяли в будущем...
#424 #318374
>>318367

>Так магниты не юзают много энергии, им немного тока подавай, чтобы они магнитили.


нет.
#425 #318375
Какая на самом деле освещённость на Марсе днём? Она ведь не сравнима с днём на Земле? Там по идее даже в полдень должен стоять полумрак из-за большей удалённности от Солнца.
318378318381
116 Кб, 1041x775
228 Кб, 1920x1080
#426 #318378
>>318375

>Какая на самом деле освещённость на Марсе днём? Она ведь не сравнима с днём на Земле? Там по идее даже в полдень должен стоять полумрак из-за большей удалённности от Солнца.



Вопросы?
318379318383
#427 #318379
>>318378
А есть фоточки с видимым размером солнца?
318380
#429 #318381
>>318375
Если песчаной бури нет, то вполне себе прекрасно.

>Там по идее даже в полдень должен стоять полумрак из-за большей удалённности от Солнца.


С хуя ли? Марс не так далеко, даже на Плутоне достаточно света, что бы, например, комфортно читать книгу.
318382
#430 #318382
>>318381

>даже на Плутоне достаточно света, что бы, например, комфортно читать книгу


Ебанутый? На Плутоне Солнце ненамного ярче полярной звезды.
318384318391
#431 #318383
>>318378
Солнце для наблюдателя с Марса будет размером в 0,625 размера, видимого с Земли (0.35°), и поставляет всего 40 % энергии, имея яркость приблизительно как во время слегка облачного земного полудня.
#432 #318384
>>318382
Пиздуй нахуй, ньюфаг, прежде чем блеять свои домыслы сходи в гугл.

>На Плутоне Солнце ненамного ярче полярной звезды


Охуительные истории.
318386
#433 #318386
>>318384
Нюфаня, на олдфага ты никак не тянешь. Иди собирай портфель.
318387
#435 #318388
>>318387

>You can see for yourself how much light there is on Pluto. If you go outside on a clear day a few minutes after sunset or a few minutes before sunrise, that's about how much sunlight there is on Pluto at noon.


Портфель-автобус-школа
318394
#436 #318391
>>318382
Ты ебанат?
#437 #318394
>>318388

>Выдали ссылку, где уебку по его мухосрани можно сравнить освещенность. Уебок продолжает блеять про кромешную темноту и подобное говно.

#438 #318491
А углеродные нанотрубки для лифта какой длинны должны быть? Только не говорите что всю длину.
318494
#439 #318494
>>318491
они должны быть 100%-ой чистоты, что есть маняфантазия, по теперешней технологии
318510
#440 #318510
>>318494
Т.е. куча нанотрубок длиной 35 000 км?
164 Кб, 1400x345
#441 #318537
У меня есть вопросы.
1)Существуют ли или возможно ли создать растения, которые смогут выжить на марсе?
2)Возможно ли вернуть атмосферу на марс чтоб там человек жить мог? Откуда её взять?
2.1)А если и возможно, то как сделать, чтоб она обратно не исчезла?
318540318563
#442 #318540
>>318537

> А если и возможно, то как сделать, чтоб она обратно не исчезла?


магнитосферу надо
астероид там на орбиту повесить или проволоки намотать и ток пустить
318542
#443 #318542
>>318540

>астероид на орбиту повесить


Но это же невозможно. От слова "совсем".
Там столько топлива надо будет, что... невозможно.
318544
#444 #318543

>Существуют ли или возможно ли создать растения, которые смогут выжить на марсе


Дядь, какие растения при -125 градусах цельсия, а? Ты ёбнутый штоль?
Воды нет, атмосферы как таковой нет, плодородного слоя почвы нет (откуда ему взятся?)
Растения по твоему голые камни жрут или святым духом питаются?
318548
#445 #318544
>>318542
ещё есть вариант погасить Солнце,лел
#446 #318548
>>318543

>Воды нет


Есть. Она намерзает на солнечных панелях.

>атмосферы как таковой нет


Какая-то есть. Хватает на пылевые бури даже.

>плодородного слоя почвы нет


Разве нет? Я не в курсе, но мне кажется всякие питательные вещества для растений должны там быть. Минералы всякие, или чё там растениям надо. Или минералы это органика какая-нибудь?

>Растения по твоему голые камни жрут


Мне кажется да, если минералы в почве это просто камни.
318549
#447 #318549
>>318548
бля, ты понимаешь, что ты порождение научпопа, который искажает факты чтобы воодушевлять школоту? Какая там вода нахуй? раствор хлоратов? и лед в перемешку в пылью? Атмосфера там такая, что на поверхности почти что вакуум. Ты не заметишь разницы - что в вакууме, что на Марсе. Сдохнешь с одинаковой скоростью. Плодородный слой на земле образовывался миллиарды лет нахуй. "Всякие питательные вещества" это какие? "Минералы всякие". Да, посыпем песочек на айсберг и посадим картошку. Будет урожай уже через пару месяцев.
318556318562
#448 #318550

>Разве нет? Я не в курсе


У тебя всё "не в курсе", Мичурин.
Попробуй на вершине Эвереста растения поищи, клоун. Там условия раз в 5 по мягче чем на марсе. Не забудь нам о успехах рассказать.
Растения у него камни жрут, лол. Чудо даже не соображает, что до того как любая растительная ебала, и прочая муйня вылезла из океана на сушу, эту самую сушу, те самые "голые камни" ~3 миллиарда лет жрали бактерии.
Бактерии подготовили "плацдарм" на котором потом всё и заколосилось.
318563
#449 #318556
>>318549
Ты же понимаешь, что на аналоге марсианского грунта уже вырастили нямку? Исследования продолжаются.

И пока я буду плавать с няшными тяночками в кратере-джакузи, ты будешь страдать и погибнешь, от нашествия орков бабахов жаждущих лебенсраума на перенаселенной, истерзанной ядерной войной Земле.
318583
#450 #318558
Эксперты по бабулиным патиссонам, из 5 Б, подъехали.
Все в грядки.
#451 #318562
>>318549
Да вы заебали своими перхлоратами уже! Их там совсем нихуя.
318586
#452 #318563
>>318550
То есть, по-твоему, бактерии могут жрать камень? Тогда этому >>318537 можно посоветовать заспамить планетку бактериями
#453 #318569

>Этому можно посоветовать заспамить планетку бактериями


Вам бы обоим, для начала, курс начальной школы заспамить.
Условия выживаемости органики там, влияние радиации, жесткого ультрафиолета, ебанистических минусовых температур, ноль-ноль и хуй повдоль атмосферного давления там... вот это вот всё.
318572
#454 #318572
>>318569
Хорош кудахтать. Говори, могут бактерии жрать голый камень или нет?
#455 #318583
>>318556
Это хуйня была, а не эксперименты. Куча неизвестных заменили данными, которые искажали эксперимент в сторону выгодную одному решению. Кароче хуйня. В какой-то момент истории будет реальное освоение Марса и люди столкнуться с проблемами, а сейчас за 20 лет до этого какие-то пиздуны пытаются создавать романтические картины, манямирок для школотронов.
318590
#456 #318586
>>318562

>Их там совсем нихуя.


кто сказал? Единственное НИХУЯ - это количество мест, где брали почву на анализ.
#457 #318587

>Говори, могут бактерии жрать голый камень или нет?


Ммм...не знаю как правильно на ваш петушинный переводится: "архей, археи, литотрофы". В гуголе забей кароч. Неешподумой сплошной. Будет о чём покукарекать.
#458 #318590
>>318583

>хуйня была, а не эксперименты


>школота ряяя


Ясно.
318591
#459 #318591
>>318590
Чё орем? Мы тут разбирали в спейсаче. Грунт брали сугубо усредненный, апроксимативный. Потому что мы не знаем что там за грунт. Брали несколько совочков на небольшую глубину из нескольких мест со всей планеты. Этого нихуя не достаточно.
А эти эксперименты часто проводятся с четкой аджендой - генерировать позитив. Потому что если бы у них не получилось вырастить растения в марсианском грунте, то об этом не написали бы новостные сайты. О нем, не написали бы. Мы как ученые знали бы, что это научные данные, а они как студенты бы воспринимали бы это как личную неудачу. Потому грунт был без всего того, что может быть в почве Марса вредного для растения и с примесями гипотетического того, что там может быть полезно для растений.
#460 #318596
Если к нам в солнечную систему прилетят замороженные инопланетяне вместе со своей планетой (которые скажем, удрали от своей умирающей звезды), где и как им лучше припарковаться, чтобы растаять, не сгореть, не быть сожранным Юпитером и не послать сраную земляшку по эллиптической кривой в местное светило?
318599318603
#461 #318599
>>318596
Нигде. Постороннее тело планетарной массы легко может распидорасить устойчивость СС, и не только земляшку. (на самом деле никто не знает, устойчива ли она вообще)
#462 #318603
>>318596
Хотелось бы сказать, что их место возле параши в облаке Оорта, но если эти чуханы привезли с собой целую планету, то, боюсь, нашего разрешения им не потребуется.
318605
#463 #318605
>>318603

>что их место возле параши в облаке Оорта


Да ну в пизду, что бы миллиарды комков грязного снега устремились во внутреннюю часть солнечной системы? Спасибо, не надо.
318606
#464 #318606
>>318605
Охуенно было бы, может на марс воду завезли.
318608
#465 #318608
>>318606
>>318606
На Марсе воды - дохуища, нахрена там еще? Что бы она так же замерзла?
#466 #318620
Позволяют ли современные технологии создать полноценную космическую станцию с защитой от радиации земного уровня, крутящимися жилыми отсеками и цехами для сборки и ремонта? Такая станция могла бы быть портом для кораблей, которые приносили бы богатые топливом и материалами астероиды.
318621
#467 #318621
>>318620
Ну, скажем так, если бы внезапно появились задачи на все йобистости, которые ты запросил, то лет за 15-20 могли бы соорудить при ощутимом интересе со стороны частных или государственных инвесторов. А может и быстрее. Но, задач как бы нет.
#468 #318637
>>318106
мы получили 1 кг материи и 1 кг антиматерии. Чтоб антиматерия выдала энергию, нужно чтоб она столкнулась с материей. То есть на 1 кг антиметерии нужен 1 кг материи. А ты из метерии уже дом построил. Хули ты будешь делать теперь?
#469 #318660
Прям щас видно очень хорошо какую-то планету. Только какую? Венеру, Марс?
318666
#470 #318664
Чет не найду тредов по самодельным ракетам. Это у вас тут не обсуждается?
318665318667
#471 #318665
>>318664
Обсуждается, было несколько тредов и в тупых вопросах ветки, только никто не делает толком, в основном всякие омские КБ и прочее подобное.
#472 #318666
>>318660
Это смотря где ты живешь. Венера ближе к горизонту, Марс немного повыше.
#473 #318667
>>318664
Не искал потому что. Но увлечение не популярное на постсоветском пространстве, так что имеем, что имеем.
https://2ch.hk/spc/res/215603.html (М)
https://2ch.hk/spc/res/318217.html (М)
#474 #318670
Могут ли быть галактики такой большой средней плотности, что излучают с большим грав. красным смещением или даже почти чёрные дыры?
318758
194 Кб, 1391x600
189 Кб, 744x686
#475 #318673
Объясните тупому второкласснику, плиз.

1.Планеты, вращающиеся вокруг Солнца, являются спутниками Солнца?

2. После того, как летательный аппарат вылетает за пределы тяготения Земли, и попадает в открытый космос, он начинает вращаться по орбите вокруг Солнца или волен сразу покинуть пределы Солнечной системы? Для того, чтобы вылететь за пределы Солнечной системы с Земли, нужно с Земли запустить аппарат со второй космической скоростью для Солнца, 618 км\с, или с третьей космической скоростью 16,6 км\с, в чем разница? Короче обхясните на пальцах, как вылететь за пределы СС, если можно.

3.Бля, даже не знаю как сформулировать. Почему вообще существуют эллиптические орбиты, ведь в одних точках орбиты тело гораздо дальше уходит от эксцентриситета, почему оно вообще "возвращается" обратно на близкое расстояние?
Отсюда же другое непонимание мое растет - как далеко можно удалиться от Солнца, оставшись при этом на его орбите? И где та грань, где Солнце еще притягивает, а вот тут уже нет? Рисунок 1.

4.Рисунок 2. Могу ли я свободно двигаться внутри солнечной системы по такой вот траектории на своем корабле или я обязательно буду вращаться вокруг какого-нибудь говна? С какой скоростью мне надо летать, чтоб не тупо кружиться вокруг Солнца?

Если объединить эти вопросы в один большой и тупой, то он звучит так. При каких условиях, я буду летать как хочу, а не просто вращаться вокруг чего-нибудь по какой-нибудь орбите?
318674318675
#476 #318674
>>318673
1.da
2.первое, 16.6, а 618 - это если стартовать на поверхности солнца. т.е. если б солнце было размером с орбиту Земли, но той же массы, то нужно было набрать скорость (вертикальную) больше 16.6, чтоб улететь от него бесконечно далеко.
3. Чем дальше, тем ниже скорость отдаления, медленнее уменьшение гравитации и дольше время её воздействия. Солнце притягивает отовсюду в пределах 4.6лярдов световых лет (с момента его образования), но гравитационное влияние уравнивается со всеми остальными звёздами где-то на полпути к ближайшей звезде (2 световых года), т.е. если поместить на таком расстоянии тело в состоянии покоя относительно солнца, то оно медленно тронется с места и полетит хз куда.
4.Лети куда хочешь, если топливо есть, но космокорабли дальнего следования разгоняются за счёт гравитационных манёвров.
318676318677
#477 #318675
>>318673

>Планеты, вращающиеся вокруг Солнца, являются спутниками Солнца?


Да, являются

>После того, как летательный аппарат вылетает за пределы тяготения Земли, и попадает в открытый космос, он начинает вращаться по орбите вокруг Солнца или волен сразу покинуть пределы Солнечной системы?


Да, на орбите вокруг Солнца. Но не забывай, что практически все планеты (особенно жирный Юпитер) влияют на траекторию КА.

>Для того, чтобы вылететь за пределы Солнечной системы с Земли, нужно с Земли запустить аппарат со второй космической скоростью для Солнца, 618 км\с, или с третьей космической скоростью 16,6 км\с, в чем разница?


Космические скорости для Солнца это если бы ты стартовал с его поверхности. Так что 16,6 км\с. Сюда ведь плюсуется орбитальная скорость Земли.

>Почему вообще существуют эллиптические орбиты, ведь в одних точках орбиты тело гораздо дальше уходит от эксцентриситета, почему оно вообще "возвращается" обратно на близкое расстояние?


Гугли "Задача двух тел", получишь примерное представление. Осторожно, матан! Вкратце - так действуют закон всемирного тяготения. В данном случае вполне достаточно уравнения дедушки Ньютона: F = G x (M1xM2)/R2

>Могу ли я свободно двигаться внутри солнечной системы по такой вот траектории на своем корабле или я обязательно буду вращаться вокруг какого-нибудь говна?


Ты в любом случае будешь на орбите (читай - в сфере влияния) какого-либо небесного тела. Другой вопрос в том, будет ли это замкнутая орбита или же гиперболическая. Представь, что ты пиздюхаешь с одного конца Солнечной системы на другой по большой такой дуге в облёт Солнца. Это будет гиперболическая, то есть, не замкнутая орбита.

>При каких условиях, я буду летать как хочу, а не просто вращаться вокруг чего-нибудь по какой-нибудь орбите?


При условии, что у тебя будет достаточно большой запас dV для пиздюхания "куда хочу". Но ты всё равно будешь на орбите, даже если будешь лететь по прямой, почему - читай выше.
318677
49 Кб, 604x604
#478 #318676
>>318674
Блин, анончик, огромное тебе спасибо,многое прояснил.

Ответишь еще на пару идиотских вопросов?

1.Как мне "упасть" с орбиты тела обратно на космическое тело?

2.Почему вообще тела выходят на орбиту, а не притягиваются в лепешку к телу? Вот, например, летел какой-нибудь астероид мимо, попал под "гравитационное влияние" (сори за словарь) солнца, стал притягиваться, притягиваться, пока не упал на поверхность, как подброшенный мячик падает обратно на землю. Или так и происходит просто ооооочень медленно, что мы это не фильтруем даже?

3.Искусственные спутники земли просто висят или могут немного менять скорость движения по орбите, если им нужно?
318678
#479 #318677
>>318675
>>318674
Вы лучшие:3
#480 #318678
>>318676

>Как мне "упасть" с орбиты тела обратно на космическое тело?


Опустить нижнюю точку орбиты в тело. Или если у тела есть атмосфера - в атмосферу.

>Почему вообще тела выходят на орбиту, а не притягиваются в лепешку к телу?


Почему ты не притягиваешься к Земле в лепёшку, а ходишь и прыгаешь? И в случае орбиты и в случае прыжка с крыши действуют абсолютно те же законы.

>Или так и происходит просто ооооочень медленно, что мы это не фильтруем даже?


Ну да. Несмотря на ебические орбитальные скорости все эти движения происходят довольно медленно. Если я правильно понял твой вопрос, конечно.

>Искусственные спутники Земли просто висят или могут немного менять скорость движения по орбите, если им нужно?


Зависит от орбиты, на которой находится КА. На низкой орбите (например, 400 км, как МКС), спутник летит с первой космической скоростью. С точки зрения земного наблюдателя - довольно быстро пролетающая яркая точка в небе. На геосинхронной (или геостационарной, как тебе больше нравится, хотя тут тоже есть свои тонкости) спутник тоже двигается, но с точки зрения земного наблюдателя "висит" над одной точкой поверхности. На самом деле выписывает восьмёрки, но это неважно. Также зависит от конструкции КА, разумеется. Как правило у них есть небольшой запас топлива для коррекции орбиты.
318679
#481 #318679
>>318678
Я имею ввиду, почему ракете для того, чтобы оторваться от поверхности (например Земли) все же нужно приложить какую-то силу, как и мне для прыжка в высоту, но по достижении какой-то высоты двигатели можно выключить и висеть себе над подверхностью, не падая обратно? Что за магическая высота, что вот здесь метр ниже ты еще притянешься к поверхности, а вот здесь метром выше уже нет? Я-то в любом случае после прыжка приземлюсь на поверхность, а не стану вращаться по орбите в метре над поверхностью.

Простите за мою гуманитарность, не гоните ссаными тряпками, хочу хоть что-то понять.
318680
53 Кб, 416x450
#482 #318680
>>318679
Вся мякотка не в высоте, а в скорости, мой гуманитарный друг.
Давай на минутку представим, что мы на Луне, чтобы не мешала атмосфера, кидаем мяч с пика Гюйгенса. Ты его швыряешь, мяч летит какое-то расстояние по дуге и падает. Чем сильнее кидаешь - тем дальше летит и тем более пологая дуга. И в какой-то момент его скорость становится достаточной для того, чтобы никогда не упасть. Он сделает оборот вокруг Луны и прилетит с противоположной стороны.
На Земле этот фокус очень мешает проделать атмосфера, поэтому кроме скорости нас волнует ещё и высота. Если на Луне достаточно лететь на пару километров выше лунных гор, то на Земле, как ты понимаешь, это не прокатит. Пикрелейтед - классическая иллюстрация из учебников астрономии. А может, даже и физики.
318682
#483 #318681
Теперь моя очередь задавать тупые вопросы.
Спейсаны, как лучше рассчитать время путешествия от Земли к Марсу строго с ускорением 1g - полпути разгоняемся, полпути тормозим? На готовые расчёты просьба ссылок не давать, хочу сам понять, как это делается.
Стартуем с низкой орбиты, прилетаем тоже на низкую. Если получится учесть ещё и движение Марса, будет просто охуенно.
318736
7 Кб, 200x193
#484 #318682
>>318680
Т.е. если абстрагироваться и представить Землю как идеально гладкий шар без атмосферы, на котором стоит метровый табурет, на котором стоит метатель копья, и он метает это копье параллельно поверхности земли с первой космической скоростью, то это копье облетит землю в метре над ее поверхностью и проткнет стоящего на табурете метателя в спину и потом упадет, потому что уменьшило скорость?
318683
#485 #318683
>>318682
Абсолютно верно
318684
#486 #318684
>>318683

А это принципиально, куда должен быть направлен вектор скорости или нет? Просто чем перпендикулярнее поверхности, тем выше орбита, или как?

скажи, как заебу
318685
7 Кб, 364x150
#487 #318685
>>318684
Тут всё несколько сложнее и на пальцах я не объясню. Не потому, что вопрос очень сложный, а потому что я не могу объяснить просто. Но могу дать совет - скачай Kerbal Space Program и хотя бы пройди обучение, гарантирую - ты многое поймёшь.
Что значит - заебал? Тяга к знаниям крайне похвальна. И не бывает тупых вопросов, бывают тупые ответы. Трипл не пиздит:)
318686
506 Кб, 1600x1200
#488 #318686
>>318685
Хорошо, спасибо.

И последнее, я правильно понимаю, что поднять на нужную высоту (орбиту) можно с любой скоростью, гораздо меньшей чем 8км/с, главное уже на нужной высоте задать первую космическую и затем можно выключать двигатели? Потому что вот смотрю я всякие запуски, и что-то я не вижу, чтобы они стартовали со скоростью 800 метров в секунду.
318688318705
#489 #318688
>>318686
Лол, кербонавты
Скорость обычно набирается постепенно, но в целом принцип верен.
318689
14 Кб, 411x412
#490 #318689
>>318688
Огромное мерси. Теперь можно и спать лечь наконец.
#491 #318692
Космоплан вроде DynaSoar + пара бустеров от Фалкона-9 под брюхом. Какие подводные камни?
318693
#492 #318693
>>318692
В смысле, он их несёт? А куда?
318694318696
#493 #318694
>>318693
и нахуя?
#494 #318696
>>318693
Они его несут. Идея в том, что ПейсХ никак не могут придумать, как вторую ступень спасти.
318698318699
#495 #318698
>>318696

> ПейсХ никак не могут придумать, как вторую ступень спасти


ниет
#496 #318699
>>318696
А, ты об этом. Ну собственно никаких подводных камней особо. Динозавр и Спираль такими и предполагались - в пердаке лаптя ступень, потом он возвращается. Такими делают X-37B и Дримчейзер.

Но раз уж речь о пейсх, они другим образом собрались возвращать вторую ступень - складной или частичный теплощит вокруг движка и полное VTVL, а не VTHL, посмотри их маркетинговый ролик от 2011 кажется. Так легче по массе. У них проблема не в самом способе, а в том, что для любого из них не хватает энергетики фалкон 9, при сохранении каких-либо полезных функций.
#497 #318700
Можно ли незаметно доставить ровер на Луну?
318701318702
#498 #318701
>>318700
Нет. Всё ведётся от запуска до посадки. На обратной стороне только не увидят посадку, но после ближайшего пролета LRO все будет видно.
#499 #318702
>>318700
тока если не с землюшки, лал
#500 #318704
>>317031
Поэтому в бассейне и на глубине 10ти метров и отыгрывают невесомость, только вместо обтягивающего водолазного костюма скафандр внутри которого какое-то давление, интересно атмосферное или нет или такое при котором создаются какие-то особые процентные соотношения окружающего давления и внутреннего
318716318736
#501 #318705
>>318686
Если не будет трения об атмосферу т.е. не будет атмосферы вообще то неважна альтитуда полета тела по отношению к принятой нулевой отметки Т.е. на луне достаточно не задевать верхушки кратеров и не поднимать пыль с поверхности, чтобы она не тормозила аппарат или снаряд и скорость и высота не будет меняться.
Думаю еще в долгосрочной перспективе важно, чтобы гравитационная константа планеты на траектории движения была постоянна иначе тело упадет или вылетит из гравитации планеты
вроде же все орбиты у нас идут вдоль экватора по синусоиде%%
318707
#502 #318707
>>318705

> гравитационная константа


ты хотел сказать масса
318709
#503 #318709
>>318707

Же которая у Земли 9,81
%%На полюсе меньше, на экваторе больше, поэтому если запустить объект на полюце, пролетая через экватор он будет тормозиться гравитацией и упадет, а если запустить на экваторе с достаточной скоростью относительно экваториального ЖЕ то на полюсе объект будет отдаляться от орбиты и уйдет в открытый космос
318710318711318736
#504 #318710
>>318709
это не гравитационная константа, это УСП
318711
#505 #318711
>>318709

Бля или наоборот, да наоборот, на полюсе g больше позорный самофикс

>>318710
Ну я децл туповат
#506 #318716
>>318704
в бассейне на глубине 10 метров будет давление 11 атмосфер (с каждым метром глубины давление воды растет на одну атмосферу). это давление легко переносится, и даже не особенно физически подготовленный человек при некотором навыке ныряния может там находиться достаточно спокойно.

скафандр надувают скорее всего на те же условных 11 атмосфер (в космосе он должен выдерживать разницу между внешним и внутренним давлением всего в 1 атмосферу, под водой избыточно надувать нет смысла, чтобы не увеличивать плавучесть)

на определенной глубине тело будет иметь нулевую плавучесть. выше этой глубины плавучесть положительная, и скафандр будет выплевывать на поверхность, ниже - будет тонуть. при нулевой (нейтральной) плавучести, не считая давления и сопротивления воды, ощущения будут максимально близки к невесомости (выталкивающая сила сравнивается с притяжением земли).

скорее всего проектируют этот тренировочный комплекс с заведомо положительной плавучестью скафандра, и на него навешивают грузы + вес пассажира, чтобы любой человек с любым весом в этом скафандре на рабочей глубине висел в воде и не тонул/не всплывал.
318743
#507 #318725
Поясните, чем токамаки отличаются по сути от ИТЕР?
Почему сейчас не делают токамаки?
318728
#508 #318728
>>318725
ебу дал?
ITER и есть токамак
318730
#509 #318730
>>318728
Да? Я читал что они в чём-то кардинально отличаются.
Тогда зачем все эти Итеры и прочие эксперементальные установки, если есть уже готовое практически решение?
318732
#510 #318732
>>318730

> Я читал что они в чём-то кардинально отличаются.


мощностью, лал
4 Кб, 112x134
6 Кб, 229x87
48 Кб, 350x216
#511 #318736
>>318681
Это очень влажная интерпретация. Однако по сути есть базовые формулы кинематики, по которым можно рассчитать модели равноускоренного движения. Максимальное сближение Марса с Землей разделяет планеты расстоянием в ~0,4 а. е., или ~60 млн км. По первой формуле неизвестной величиной у нас является время. Итого имеем t=(2S/g)^1/2=(60 000 000 000/10)^1/2=~77 500с. То есть при постоянном ускорении в 1g половина пути до Марса в 30 млн км будет пройдена менее чем за сутки.
Собственно, несмотря на возможно кажущуюся неочевидность, процесс торможения займет те же ~77 500с, или 21,5 часа. Итого полет на Марс с ускорением и торможением на 1g займет немногим более 43-х часов при орбитальном сближении планет. И какое такое нахуй движение Марса ты собрался учитывать, когда полноценный расчет с тонкостями небесной механики и гравитационного влияния массивных тел системы займет несколько страниц без пояснений. Да еще и с учетом того, что для этого расчета потребуются характеристики и параметры летающей с конским ускорением вундервафли, которая пока существует только в сознании недалеких романтичных покорителей космоса.
>>318704
Тренировки в бассейне никак не связаны с разницей давлений, о чем вы? Идея в том, чтобы добиться "нейтральной плавучести", т. е. равновесия между силой тяжести и силой Архимеда. Это как бы дешевый, но менее наглядный из-за высокого сопротивления среды способ имитировать равновесие сил, действующих на тело в пространстве, чем американские горки на самолетах.
>>318709

>поэтому если запустить объект на полюсе, пролетая через экватор он будет тормозиться гравитацией и упадет


Как-то ты совсем плохо разбираешься в орбитальной механике. Начать хотя бы с того, что гравитационное поле Земли неравномерно. На полюсах g выше, да, во-первых из-за меньшего расстояния до центра масс планеты, во-вторых из-за околонулевого значения центробежной силы, хотя первое есть следствие второго. Однако при этом поле земного тяготения вообще неравномерно по планете. Из-за геологических особенностей в первую очередь. Миллигал - величина, приведенная на пике, соответственно 1/1000 Гал. Гал - единица измерения g, названная в честь Галилея, равна 1см/с^2. То есть привычное значение g=9.81 м/с^2 в галах равно g=981 Гал. В общем миллигал - величина незначительная, организмом неощутимая, требующая качественных приборов, называемых гравиметрами, для измерения. Чтобы не увлекаться зачитыванием курса гравиметрии скажу, что на любой орбите вокруг Земли каждый спутник испытывает возмущения, связанные с неравномерностью поля притяжения. Для этого в большинстве из них присутствует запас топлива для корректировки орбиты. Но не меньшее влияние на отклонения оказывают и крохи атмосферы. Так что стабильный спутник, чтобы он сам по себе летал на одной высоте десятки лет, на низкой орбите сейчас организовать не возможно. Поэтому ничто не мешает запускать спутники по полярным орбитам, просто с соответствующими поправками и запасами топлива для корректировки. Они собственно вполне себе летают.
https://ru.wikipedia.org/wiki/Полярная_орбита
4 Кб, 112x134
6 Кб, 229x87
48 Кб, 350x216
#511 #318736
>>318681
Это очень влажная интерпретация. Однако по сути есть базовые формулы кинематики, по которым можно рассчитать модели равноускоренного движения. Максимальное сближение Марса с Землей разделяет планеты расстоянием в ~0,4 а. е., или ~60 млн км. По первой формуле неизвестной величиной у нас является время. Итого имеем t=(2S/g)^1/2=(60 000 000 000/10)^1/2=~77 500с. То есть при постоянном ускорении в 1g половина пути до Марса в 30 млн км будет пройдена менее чем за сутки.
Собственно, несмотря на возможно кажущуюся неочевидность, процесс торможения займет те же ~77 500с, или 21,5 часа. Итого полет на Марс с ускорением и торможением на 1g займет немногим более 43-х часов при орбитальном сближении планет. И какое такое нахуй движение Марса ты собрался учитывать, когда полноценный расчет с тонкостями небесной механики и гравитационного влияния массивных тел системы займет несколько страниц без пояснений. Да еще и с учетом того, что для этого расчета потребуются характеристики и параметры летающей с конским ускорением вундервафли, которая пока существует только в сознании недалеких романтичных покорителей космоса.
>>318704
Тренировки в бассейне никак не связаны с разницей давлений, о чем вы? Идея в том, чтобы добиться "нейтральной плавучести", т. е. равновесия между силой тяжести и силой Архимеда. Это как бы дешевый, но менее наглядный из-за высокого сопротивления среды способ имитировать равновесие сил, действующих на тело в пространстве, чем американские горки на самолетах.
>>318709

>поэтому если запустить объект на полюсе, пролетая через экватор он будет тормозиться гравитацией и упадет


Как-то ты совсем плохо разбираешься в орбитальной механике. Начать хотя бы с того, что гравитационное поле Земли неравномерно. На полюсах g выше, да, во-первых из-за меньшего расстояния до центра масс планеты, во-вторых из-за околонулевого значения центробежной силы, хотя первое есть следствие второго. Однако при этом поле земного тяготения вообще неравномерно по планете. Из-за геологических особенностей в первую очередь. Миллигал - величина, приведенная на пике, соответственно 1/1000 Гал. Гал - единица измерения g, названная в честь Галилея, равна 1см/с^2. То есть привычное значение g=9.81 м/с^2 в галах равно g=981 Гал. В общем миллигал - величина незначительная, организмом неощутимая, требующая качественных приборов, называемых гравиметрами, для измерения. Чтобы не увлекаться зачитыванием курса гравиметрии скажу, что на любой орбите вокруг Земли каждый спутник испытывает возмущения, связанные с неравномерностью поля притяжения. Для этого в большинстве из них присутствует запас топлива для корректировки орбиты. Но не меньшее влияние на отклонения оказывают и крохи атмосферы. Так что стабильный спутник, чтобы он сам по себе летал на одной высоте десятки лет, на низкой орбите сейчас организовать не возможно. Поэтому ничто не мешает запускать спутники по полярным орбитам, просто с соответствующими поправками и запасами топлива для корректировки. Они собственно вполне себе летают.
https://ru.wikipedia.org/wiki/Полярная_орбита
318740
#512 #318740
>>318736

>Это очень влажная интерпретация.


В каком смысле влажная? Спасибо за ответ, но хуями крыть было не обязательно. Я не знал, что получится "долететь" так быстро и учитывать увеличение расстояния не понадобится.
#513 #318743
>>318716

> с каждым метром давление растёт на атмосферу


Ты наебался на порядок, физик мамкин.
#514 #318746
Аноны, не так давно где-то скачивал книжку, где популярно разъяснялась теория космических полетов. Книжка еще советская - отсканированный пдфник. Но я его куда-то проебал. И даже название забыл. Никто мне подскажет где ее можно скачать?
319762
#515 #318758
>>318670

>излучают с большим грав. красным смещением


Даже очень плотные шаровые скопления и галактические центры очень просторные штуки, что бы иметь хоть сколько-нибудь большое гравитационное красное смещение. Очень сильная зависимость от радиуса, так что даже при большой массе вклад никудышный.
ПЕРЕКОТ #516 #318775
#517 #319762
>>318746
Левантовский
Тред утонул или удален.
Это копия, сохраненная 16 июля 2017 года.

Скачать тред: только с превью, с превью и прикрепленными файлами.
Второй вариант может долго скачиваться. Файлы будут только в живых или недавно утонувших тредах. Подробнее

Если вам полезен архив М.Двача, пожертвуйте на оплату сервера.
« /spc/В начало тредаВеб-версияНастройки
/a//b//mu//s//vg/Все доски